You are on page 1of 17

DEDUCTIONS AND EXEMPTIONS; DEDUCTIONS IN GENERAL

ZAMORA v. COLLECTOR [G.R. No. L-15290. May 31, 1963.]


FACTS: Mariano Zamora, owner of the Bay View Hotel and Farmacia Zamora, Manila, filed
his income tax returns. The Collector of Internal Revenue found that the promotion
expenses incurred by his wife for the promotion of the Bay View Hotel and Farmacia
Zamora were not allowable deductions. Mariano Zamora contends that the whole amount
of the promotion expenses in his income tax returns, should be allowed and not merely
one-half of it, on the ground that, while not all the itemized expenses are supported by
receipts, the absence of some
supporting receipts has been sufficiently and satisfactorily established.
ISSUE: In the absence of receipts, WON to allow as deduction all or merely one-half of the
promotion expenses of Mrs. Zamora claimed in Mariano Zamora's income tax returns
HELD: One-half only. Claims for the deduction of promotion expenses r entertainment
expenses must also be substantiated or supported by record showing in detail the amount
and nature of the expense incurred. Considering that the application of Mrs. Zamora for
dollar allocation shows that she went abroad on a combined medical and business trip, not
all of her expenses came under the category of ordinary and necessary expenses; part
thereof constituted her personal expenses. There having been no means by which to
ascertain which expense was
incurred by her in connection with the business of Mariano Zamora and which was incurred
for her personal benefit, the Collector and the CTA in their decisions, considered 50% of the
said amount as business expense and the other 50%, as her personal expenses. While in
situations like the present, absolute certainty is usually not possible, the CTA should make
as close an approximate as it can, bearing heavily, if it chooses, upon the taxpayer whose
inexactness is of his own making.
ESSO STANDARD v. CIR [G.R. Nos. 28508-9. July 7, 1989.]
FACTS: Petitioner ESSO claimed as ordinary and necessary expenses in the same return
the margin fees it paid to the Central Bank on its profit remittances to its New York head
office.
ISSUE: WON the margin fees were deductible from gross income either as a tax or as an
ordinary and necessary business expense
HELD: Neither. The margin fees were imposed by the State in the exercise of its police
power and not the power of taxation. Neither are they necessary and ordinary business
expenses. To be deductible as a business expense, the expense must be paid or incurred
in carrying on a trade or business. The fees were paid for the remittance by ESSO as part
of the profits to the head office in the United States, which is already another distinct and
separate income taxpayer. Such remittance was an expenditure necessary and proper for
the conduct of its corporate affairs.
CIR v. GENERAL FOODS [G.R. No. 143672. April 24, 2003.]
FACTS: In its income tax return, respondent corporation claimed as deduction, among
other business expenses, the amount for media advertising for Tang, one of its products.
ISSUE: WON the subject media advertising expense for Tang incurred by respondent was
an ordinary and necessary expense fully deductible under the NIRC
HELD: Not deductible. Deductions for income tax purposes partake of the nature of tax
exemptions; hence, must be strictly construed. To be deductible from gross income, the
subject advertising expense must be ordinary and necessary. There being no hard and fast
rule on the
reasonableness of an advertising expense, the right to a deduction depends on a number
of factors such as but not limited to: the type and size of business in which the taxpayer is
engaged; the volume and amount of its net earnings; the nature of the expenditure itself;
the
intention of the taxpayer and the general economic conditions. The amount claimed as
media advertising expense for Tang alone was almost one-half of its total claim for
marketing expenses. Furthermore, it was almost double the amount of respondent
corporation's general and administrative expenses. The subject expense for the

advertisement of a single product is inordinately large. Said venture of respondent to


protect its brand franchise was tantamount to efforts to establish a reputation, and should
not, therefore, be considered as business expense but as capital expenditure, which
normally should be spread out over a reasonable period of time.
C.M. HOSKINS v. CIR G.R. No. L-28383. June 22, 1976.]
FACTS: Petitioner-appellant, a domestic corporation engaged in the development and
management of subdivisions, sale of subdivision lots and collection of installments due for
a fee which the real estate owners pay as compensation for each of the services rendered,
failed to pay the real estate broker's tax on its income derived from the supervision and
collection fees. Consequently, the Commissioner of Internal Revenue demanded the
payment of the percentage tax plus surcharge, contending that said income is subject to
the real estate broker's percentage tax. On the other hand, petitioner-appellant claimed that
the supervision and collection fees do not form part of its taxable gross compensation.
ISSUE: WON the supervision and collection fees received by a real estate broker are
deductible from its gross compensation
HELD: No. With respect to the collection fees, the services rendered by Hoskins in
collecting the amounts due on the sales of lots on the installment plan are incidental to its
brokerage service in selling the lots. If the broker's commissions on the cash sales of lots
are subject to the brokerage percentage tax, its commissions on installment sales should
likewise be taxable. As to the supervision fees for the development and management of the
subdivisions, which fees were paid out of the proceeds of the sales of the subdivision lots,
they, too, are subject to the
real estate broker's percentage tax. The development, management and supervision
services were necessary to bring about the sales of the lots and were inseparably linked
thereto. Hence, there is basis for holding that the operation of subdivisions is really
incidental to the main business of the broker, which is the sale of the lots on commission.
GANCAYCO v. COLLECTOR [G.R. No. L-13325. April 20, 1961.]
FACTS: Petitioner Santiago Gancayco seeks the review of a decision of the Court of Tax
Appeals, requiring him to pay deficiency income tax. The question whether the sum is due
from Gancayco as deficiency income tax hinges on the validity of his claim for deduction of
two (2)
items, namely: (a) for farming expenses; and (b) for representation expenses.
ISSUE: WON the two claimed deductions are allowable
HELD: No. In computing net income, no deduction shall be allowed in respect of any
amount paid out for new buildings or for permanent improvements, or betterments made to
increase the value of any property or estate. The cost of farm machinery, equipment and
farm building represents a capital investment and is not an allowable deduction as an item
of expense. Hence, the farming expenses allegedly incurred for clearing and developing
the farm which were necessary to place it in a productive state, were not an ordinary
expense but a capital
expenditure. Accordingly, they are not deductible. As for Gancayco's claim for
representation expenses, a fraction was disallowed. Such disallowance is justified by the
record, for, apart from the absence of receipts, invoices or vouchers of the expenditures,
petitioner could not specify the items constituting the same, or when or on whom or on what
they were incurred.
WESTERN MINOLCO v. CIR [G.R. No. L-61632. August 16, 1983.]
FACTS: Petitioner is a domestic corporation engaged in mining, particularly copper
concentrates for export mined from mineral lands. It was granted by the Securities and
Exchange Commission, under Certificate of Renewal No. R-1056, authority to borrow
money and issue
commercial papers. Pursuant to this authority, the petitioner borrowed funds from several
financial institutions and paid the corresponding 35% transaction tax due thereon.
Petitioner applied for a refund alleging that it was not liable to pay the 35% transaction tax.
ISSUE: WON the 35% transaction tax is a business tax that constitutes an allowable
deduction from gross income

HELD: No. The 35% transaction tax is imposed on interest income from commercial papers
issued in the primary money market. Being a tax on interest, it is a tax on income. The
petitioner who borrowed funds from several financial institutions by issuing commercial
papers merely
withheld the 35% transaction tax before paying to the financial institutions the interests
earned by them and later remitted the same to the respondent Commissioner of Internal
Revenue. Whatever collecting procedure is adopted does not change the nature of the tax.
Furthermore, whether or not certain taxes are on income is not necessarily determined by
their deductibility or non-deductibility from ross income. Income in the form of dividends,
capital gains on real property, shares of stock, and interests on savings in bank accounts
are incomes, yet they are not included in the gross income when income taxes are paid
because these are subject to final withholding taxes.
COMMISSIONER OF CUSTOMS VS PHILIPPINE ACETYLENE
COMPANY
Facts: Philippine Acetylene Company is engaged in the manufacture of oxygen, acetylene
and nitrogen, and packaging of liquefied petroleum gas in cylinders and tanks. It imported
from the United States a custombuilt liquefied petroleum gas tank. For the said importation,
the company
was assessed a special import tax amounting to PhP 3,683.00. The company paid the tax
under protest. Philippine Acetylene Company argues that it is exempt from the payment of
the special import tax. It cites as basis for its exemption Sec 6 of RA No.1394 which states
that special import taxes shall not be imposed on machinery, equipment, accessories and
spare parts, imported into the Philippines, for the use of industries. The company maintains
that it is an
industry as defined in Sec 6 of RA No. 1394. The Court of Tax Appeals sustained Philippine
Acetylene Companys contention and declared the latter exempt from the payment of the
special import tax.
Issue: Whether or not Philippine Acetylene Company may be considered engaged in an
industry as contemplated in Sec 6 of RA No. 1394 and, therefore, exempt from the payment
of the special import tax.
Held: Philippine Acetylene Company is not an industry as defined in Sec 6 of RA No. 1394.
To be an industry, the company must be engaged in some productive enterprise, not in
merely packaging an already finished product. The operation for which the company
employs the gas tank in question does not involve manufacturing or production. It is nothing
but packaging; the liquefied gas, when obtained from the refinery, has to be placed in some
kind of container to facilitate its transportation. When sold to consumers, it undergoes no
change or transformation, but is merely placed in smaller cylinders for convenience. The
process is certainly not
production in any sense. The decision of the CTA is reversed and Philippine Acetylene
Company is held liable for the payment of the special import tax, as it is not an industry
exempt from the payment of such tax.
COMMISSIONER OF INTERNAL REVENUE VS ARNOLDUS
CARPENTRY SHOP, INC.
Facts: Arnoldus Carpentry Shop, Inc. is a domestic corporation engaged in the business of
preparing, processing, buying, selling, exporting, importing, manufacturing, trading and
dealing in cabinet shop products, wood and metal home and office furniture, cabinets,
doors, windows,
etc., including their component parts and materials, of any and all nature and description.
The Commissioner of Internal Revenue conducted an investigation of the business tax
liabilities of Arnoldus Carpentry Shop, Inc. After the examination, the CIR concluded that
Arnoldus Carpentry Shop, Inc. is an independent contractor under Sec 205 (16) [now Sec
169 (q)] of the Tax Code. As a result of this classification, Arnoldus Carpentry Shop, Inc.
was assessed deficiency tax (PhP 88,972.23) plus charges and interest. This tax deficiency
was a consequence of the 3% tax imposed on the companys gross export sales which, in
turn, resulted from the CIRs finding that categorized the company as a contractor. Arnoldus
Carpentry Shop, Inc. protested the assessment maintaining that it is a manufacturer and

therefore entitled to tax exemption on its gross export sales under Sec 202 (e) of the
National Internal Revenue Code. The CIR stood by its initial finding that Arnoldus Carpentry
Shop, Inc. is a contractor, not a manufacturer. Arnoldus Carpentry Shop, Inc. appealed to
the Court of Tax Appeals. The CTA held that Arnoldus Carpentry Shop, Inc. is a
manufacturer, effectively reversing the decision of the CIR.
Issue: Whether Arnoldus Carpentry Shop, Inc. is a manufacturer or contractor. If found a
manufacturer, the company is, therefore, not liable for the amount assessed as deficiency
contractors tax.
Held: Arnoldus Carpentry Shop, Inc. is a manufacturer as defined in the Tax Code and not
a contractor. A contractor under Sec 205 (16) [now Sec 170 (q)] of the Tax Code is one
whose activity consists essentially of the sale of all kinds of services. The business of
Arnoldus Carpentry Shop, Inc. does not fall under this definition. The company sells goods
which it keeps in stock and not services. On the other hand, a manufacturer, under Sec 187
(x) [now Sec 157 (x)] of the Tax Code, is one who by physical or chemical process alters
the exterior texture or
form or inner substance of any raw material or manufactured or partially manufactured
product. The term manufacturer had been considered in its ordinary and general usage,
and Arnoldus Carpentry Shop, Inc. falls under this definition. The Court affirmed the
decision of the CTA holding that Arnoldus Carpentry Shop, Inc. is a manufacturer. The
company is entitled to the
tax exemption under Sec 202 (d) and (e) [now Sec 167 (d) and (e)] of the Tax Code. It is
not liable for the deficiency contractors tax assessed by the CIR.
BANK OF THE PHILIPPINE ISLANDS VS TRINIDAD
Facts: Bank of the Philippine Islands is a domestic banking corporation, operating under a
special charter granted by the Philippine Legislature through Act No. 1790. The charter
contains a clause to the effect that no law shall be made or enforced imposing a charge or
taxation upon BPI which shall not apply equally to other banks of a similar type operating
under similar conditions. The Collector of Internal Revenue collected internal revenue taxes
upon BPIs circulating notes issued by the bank for the years 1919-1921. The Philippine
Legislature, through Act No. 2612, created the Philippine National Bank. Under Sec 18 of
the PNB Charter, it is provided that the circulating notes of PNB shall be exempt from any
and all taxes. The CIR did not collect taxes upon the circulating notes of PNB because of
the exemption granted to the latter by Sec 18 of Act No. 2612. BPI contends that BPI and
PNB are banks of similar type and operating under similar conditions. Thus, BPI should
also be entitled to the same exemptions and privileges granted to PNB. Essentially, BPI
argues that it should also be exempt from the payment or taxes upon its circulating notes.
Issue: Whether or not BPI and PNB are banks of similar type and operating under similar
conditions. Whether or not BPI is exempt from the payment or taxes upon its circulating
notes
Held: The Court noted that subsequent to the filing of the action, PNB paid to the CIR PhP
519,043.03 as taxes upon its circulating notes. This development, according to the Court,
rendered it unnecessary to decide the question as to whether BPI and PNB are of similar
type or operating under similar conditions. BPI is not exempt from the payment of taxes
upon its circulating notes. It is not entitled to a refund of the payments it made for that
purpose. The
Court used Sec 1499 of the Administrative Code of 1917 in ruling that BPI is not tax
exempt. This Section allows the collection from banks of taxes on capitals, deposits, and
circulation. Exemptions are strictly construed against the taxpayer, so unless BPI can show
clearly that it has been granted the status of exemption, then it cannot avail itself of such
entitlement. The fact that one person may not have paid or been required to pay his taxes
does not exempt another from the payment of his legal taxes, or legally entitle him to a
refund of any taxes which he
has paid.
DEDUCTIONS AND EXEMPTIONS; DEDUCTIONS IN GENERAL;
ALLOCATION
YUTIVO SONS HARDWARE CO. VS COURT OF TAX APPEALS

Facts: Yutivo Sons Hardware Co. is a domestic corporation engaged in the importation and
sale of hardware supplies and equipment. It bought a number of cars and trucks from
General Motors Overseas Corporation, an American corporation licensed to do business in
the Philippines. As
importer, GM paid sales tax prescribed by sections 184, 185 and 186 of the Tax Code on
the basis of its selling price to Yutivo. Said tax being collected only once on original sales,
Yutivo paid no further sales tax on its sales to the public. Southern Motors, Inc. was
organized to engage in the business of selling cars, trucks and spare parts. Its shares were
subscribed in five equal proportions by the descendants of the founders of Yutivo. When
GM withdrew from the Philippines, the cars and trucks purchased by Yutivo from GM were
sold by Yutivo to SM which, in turn, sold them to the public in the Visayas and Mindanao.
GM appointed Yutivo as importer for the Visayas and Mindanao, and Yutivo continued its
previous arrangement of selling exclusively to SM. In the same way that GM used to pay
sales taxes based on its sales to Yutivo, the latter, as importer, paid sales tax on the basis
of its selling price to SM, and since such sales tax, as already stated, is collected only once
on original sales, SM paid no sales tax on its sales to the public. Yutivo was investigated by
the CIR and was assessed deficiency sales
tax plus surcharge. The CIR claimed that the taxable sales were the retail sales by SM to
the public and not the sales at wholesale made by Yutivo to the latter inasmuch as SM and
Yutivo were one and the same corporation, the former being the subsidiary of the latter.
Yutivo alleged the following before the Court of Tax Appeals: (1) that there is no valid
ground to disregard the corporate personality of SM and to hold that it is an adjunct of
petitioner Yutivo; (2) that assuming that the separate personality of SM may be
disregarded, the sales tax already
paid by Yutivo should first be deducted from the selling price of SM in computing the sales
tax due on each vehicle; and (3) that the surcharge has been erroneously imposed by the
CIR. The CTA ruled in favor of CIR and ruled that the creation of SM is for Yutivo to evade
taxes, as it is
owned and controlled by Yutivo and is a mere subsidiary, branch, adjunct conduit,
instrumentality or alter ego of the latter.
Issue: Whether or not SM has a personality separate and distinct from Yutivo.
Held: It is an elementary and fundamental principle of corporation law that a corporation is
an entity separate and distinct from its stockholders and from other corporations to which it
may be connected. However, "when the notion of legal entity is used to defeat public
convenience,
justify wrong, protect fraud, or defend crime," the law will regard the corporation as an
association of persons, or in the case of two corporations merge them into one. When the
corporation is the "mere alter ego or business conduit of a person, it may be disregarded."
However, SM was not organized purposely as a tax evasion device. Moreover, it runs
counter to the fact that there was no tax to evade. The intention to minimize taxes, when
used in the context of fraud, must be proved to exist by clear and convincing evidence
amounting to more than mere preponderance, and cannot be justified by a mere
speculation. This is because fraud is never lightly to be presumed. The SC however agreed
that SM was actually owned and controlled by petitioner as to make it a mere subsidiary or
branch of the latter created for the purpose of selling the vehicles at retail and maintaining
stores for spare parts as well as service repair shops. Consideration of various other
circumstances, especially when taken together, indicates that Yutivo treated SM merely as
its department or adjunct. For one thing, the accounting system maintained by Yutivo
shows that it maintained a high degree of control over SM accounts. All transactions
between Yutivo and SM are recorded and effected by mere debit or credit entries against
the reciprocal account maintained in their respective books of accounts and indicate the
dependency of SM as branch upon Yutivo.
DEDUCTIONS AND EXEMPTIONS; ALLOWABLE DEDUCTIONS;
ORDINARY/NECESSARY BUSINESS EXPENSES
ATLAS CONSOLIDATED MINING & DEVT CORP VS
COMMISSIONER OF INTERNAL REVENUE

Facts: Atlas Consolidated Mining & Devt Corp is a corporation engaged in the mining
industry. It was assessed deficiency income tax for the year 1958 as a result of the
disallowance of certain items claimed by the company as deductions from its gross income.
Atlas claimed the following items as deductible from its gross income: (1) transfer agents
fee, (2) stockholders relation service fee, (3) US stock listing expenses, (4) suit expenses,
(5) provision for contingencies. The
Commissioner of Internal Revenue disallowed all these items. Atlas elevated the issue to
the Court of Tax Appeals. The CTA rendered a decision allowing the said items, except for
the stockholders relation service fee and the suit expenses. Both Atlas and the CIR went to
the Supreme Court to appeal the CTA decision. In GR No. L-26911, Atlas argues that the
CTA should not have disallowed the stockholders relation service fee. The corporation
contends that such fee constitutes an ordinary and necessary business expense, and
should, therefore, be allowed as a deductible expense from the companys gross income.
In GR No. L-26924, the CIR argues that the transfer agents fee and the US stock listing
fee should not have been allowed as deductions from gross income in the absence of proof
of payment for such expenses. The CIR also argues that the US stock listing expenses
should be disallowed for not being ordinary and necessary and not incurred in trade or
business, as required under Sec 30 (a) (1) of the National Internal Revenue Code. The CIR
also contends that the correct amount of disallowance for suit expenses should be PhP
17,499.98 and not PhP 6,666.65.
Issue: In GR No. L-26911, whether or not the expenses paid for the services rendered by a
public relations firm, P. K. Macker & Co., labeled as stockholders relation service fee is an
allowable deduction as business expense. In GR No. L-26924, whether or not the transfer
agents fee and the US stock listing fee should have been allowed as deduction in the
absence of proof of payments. Whether or not the US stock listing expenses should be
disallowed for not being ordinary and necessary and not incurred in trade or business.
Whether PhP 17,499.98 or PhP 6,666.65 is the correct amount of disallowance for suit
expenses.
Held: GR No. L-26911. Under Sec 30 (a) (1) of the Tax Code, three conditions have to be
complied with before a business expense is allowed as a deduction from gross income: (1)
the expense must be ordinary and necessary, (2) it must be paid or incurred within the
taxable year, and (3) it must be paid or incurred in carrying a trade or business. The Court
sustained the ruling of the CTA that the expenditure paid to P. K. Macker & Co.
denominated as stockholders relation service fee is not an ordinary expense. The fee was
paid to the PR firm as ompensation
for services carrying on the selling campaign in an effort to sell Atlas additional capital
stock. Such is not an ordinary expense because, according to the Court, expenses relating
to the recapitalization and reorganization of a corporation, the cost of obtaining stock
subscription,
promotion expenses, and commission or fees paid for the sale of stock reorganization are
capital expenditures. The stockholders relation service fee is not deductible from Atlas
gross income.
GR No. L-26924. The Court agreed with the CTA that the CIR cannot raise the issue of
payment for the first time on appeal. The fact of payment was never controverted by the
CIR during the proceedings. Failure to assert a question within a reasonable time warrants
a presumption that the party entitled to assert it either has abandoned or declined to assert
it. The Court held that the US stock listing fee is an ordinary and necessary business
expense and was correctly allowed by the CTA as a deduction. The stock listing fee is paid
annually to a stock exchange for the privilege of having Atlas stock listed. A single payment
made to the stock exchange is considered a capital expenditure (Domes Mines case).
However, payments to the stock exchange made annually or in a recurring manner are
considered ordinary and necessary business expenses (Chesapeake Corporation case).
The fees paid by Atlas partake of the latter. The Court reiterated that it is well-settled that
litigation expenses incurred in defense or protection of title are capital in nature and not
deductible. The Court sustained the CIR that the correct amount of disallowance for
litigation or suit expenses is PhP 17,499.98.
VISAYAN CEBU TERMINAL CO. INC. VS COLLECTOR OF

INTERNAL REVENUE
Facts: Visayan Cebu Terminal Co. Inc. is a corporation organized for the purpose of
handling arrastre operations in the port of Cebu. Visayan filed its income tax return for 1951
claiming the following items as deductions from the companys gross income: (1) salaries,
(2) representation
expenses (PhP 75,855.88), and (3) miscellaneous expenses. The Collector of Internal
Revenue disallowed the entire amount of representation expenses. The Court of Tax
Appeals allowed
representation expenses but set the limit at PhP 10,000.00.
Issue: Whether or not the full amount of representation expenses should be allowed as a
deduction from Visayans gross income.
Held: The Court sustained the Tax Court in holding that representation expenses fall under
the category of business expenses which are allowable deductions from gross income if
they meet the following requisites laid down in the Tax Codethey must be ordinary and
necessary expense paid or incurred in carrying on any trade or business and they must
meet the test of reasonableness in amount. The Court further agreed with the computation
made by the CTA. Because of the companys failure to provide evidence for all such
expenses (the corporation claims that the supporting papers were destroyed when the
house of the company treasurer, where the records were kept was burned), the Court
should determine from all available data the amount properly deductible as representation
expenses. The Court sustained the finding of the CTA that PhP 10,000.00 may be
considered reasonably necessary as the companys representation expenses based on
figures presented during the proceedings.
KUENZLE & STREIFF, INC. vs. THE COLLECTOR OF INTERNAL
REVENUE
Facts: Petitioner claimed as a deduction for income tax purposes for the years 1950, 1951
and 1952 salaries, directors' fees and bonuses of its non-resident president and vicepresident; bonuses of some of its resident officers and employees; and interests on earned
but unpaid
salaries and bonuses of its officers and employees. Petitioner gave to its non-resident
president and vice president for the years 1950 and 1951 bonuses equal to 133-1/2% of
their annual salaries and bonuses equal to 125 2/3% for the year 1952. Petitioner however
gave its resident officers and employees higher bonuses on the alleged reason because of
their valuable contribution to the business of the corporation which has made it possible for
it to realize huge profits during the aforesaid years. The respondent disallowed the said
deductions hence they were assessed for deficiency income taxes. Upon re-examination by
the respondents, they allowed as deductions all items comprising directors' fees and
salaries of the non-resident president and vice president, but disallowing the bonuses
insofar as they exceed the salaries of the recipients, as well as the interests on earned but
unpaid salaries and bonuses.
Issue: WON the excessive bonuses and interest should be allowed as a deduction for
income tax purposes.
Held: No. Bonuses to employees made in good faith and as additional compensation for
the services actually rendered by the employees are deductible, provided such payments,
when added to the stipulated salaries, do not exceed a reasonable compensation for the
services
rendered" Requisites for deductibility of employee bonuses: (1) the payment of the bonuses
is in fact compensation; (2) it must be for personal services actually rendered; and (3) the
bonuses, when added to the salaries, are "reasonable . . . when measured by the amount
and quality of the services performed with relation to the business of the particular
taxpayer". There is no fixed test for determining the reasonableness of a given bonus as
compensation. Deductible amount of bonuses is not limited to the amount of salary of its
recipient. The prevailing circumstances
should be considered. However In this case, the bonuses given to resident employees
were higher than its non-resident officers on the reason that the resident officers and
employees had performed their duty well and rendered efficient service. It does not
necessarily follow that they should be given greater amount of additional compensation in

the form of bonuses than what was given to the non-resident officers. The non-resident
officers had rendered the same amount of efficient personal service and contribution to
deserve equal treatment in compensation and
other emoluments with the particularity that their liberation yearly salaries had been much
smaller. Interest should also be disallowed. . Under the law, in order that interest may be
deductible, it must be paid "on indebtedness" (Section 30, (b) (1) of the National Internal
Revenue Code). It is therefore imperative to show that there is an existing indebtedness
which may be subjected to the payment of interest. Here the items involved are unclaimed
salaries and
bonus participation which in our opinion cannot constitute indebtedness within the meaning
of the law because while they constitute an obligation on the part of the corporation, it is not
the latter's fault if they remained unclaimed. The willingness of the corporation to pay
interest thereon cannot be considered a justification to warrant deduction.
ALHAMBRA CIGAR & CIGARETTE MANUFACTURING COMPANY,
petitioner-appellant, vs. THE COMMISSIONER OF INTERNAL
REVENUE, respondent-appellee.
[G.R. No. L-23226. November 28, 1967.]
Facts: The petitioner claimed as deductible expense for income tax purposes salaries,
bonuses, commissions and directors fees paid to A. P. Kuenzle and H. A. Streiff, who were
the President and Vice-President, respectively, of the petitioner. The Commissioner of
Internal Revenue
disallowed a portion of the bonus, commission and directors fees as deductions. Hence
was assessed for deficiency income tax.
Issue: WON the bonuses, directors fees and commissions are valid deductions for income
tax purposes.
Held: No. Whenever a controversy arises on the deductibility, for purposes of income tax,
of certain items for alleged compensation officers of a corporation, it is necessary to
determine whether "personal services" have been "actually rendered" by said officers, and,
in the
affirmative, what is the "reasonable allowance" therefor. As correctly held by the court of tax
appeals, The bonus paid to each of said officers was reduced to the amount equivalent to
that paid to Mr. W. Eggmann, the resident Treasurer and Manager of petitioner. Petitioner
seeks to justify the increase in the salaries of Messrs. Kuenzle and Streiff on the ground of
increased costs of living. The said officers of petitioner are, however, non-residents of the
Philippines.
As to commissions and directors' fees there is no evidence of any particular service
rendered by them to petitioner to warrant payment of commissions. There is also no
justification for the payment of the directors fees. Being non-resident President and VicePresident of Petitioner corporation of which they are the controlling stockholders, said
commissions and directors' fees, payment of which was based on a certain percentage of
the annual profits of petitioner, are in the nature of dividend distributions.
AGUINALDO INDUSTRIES CORPORATION (FISHING NETS
DIVISION) vs. COMMISSIONER OF INTERNAL REVENUE and THE
COURT OF TAX APPEALS
Facts: Upon investigation of petitioner's 1957 income tax returns of its Fish Nets Division,
the Bureau of Internal Revenue examiner found that the amount of P61,187.48 was
deducted from the gross income as additional remuneration paid to the officers of petitioner
and that such amount was taken from the net profit which petitioner derived from an
isolated transaction (sale of a parcel of its land) which is not in the course of or carrying on
of petitioner's trade or business. The examiner recommended disallowance of the
deduction, but petitioner insisted
otherwise, claiming that the payment of the allowance or bonus was pursuant to its by-laws.
The Court of Tax Appeals held the petitioner liable for deficiency income tax plus surcharge
and interest

Issue: WON the profit derived from the sale of its land is tax-exempt income under
Republic Act No. 901
Held: No. Petitioner may not raise the question of tax exemption for the first time on review
where such question was not raised at the administrative forum
Issue: WON the bonus given to the officers of petitioner as share in the profit realized from
the sale of the land is deductible expense for tax purposes
Held: No. The bonus given should be considered as deductible for income tax purposes
only if payment was made for service actually rendered and it is reasonable and necessary.
The records show that the sale was effected through a broker who was paid by petitioner a
commission for his services. On the other hand, there is absolutely no evidence of any
service actually rendered by petitioner's officers which could be the basis of a grant to them
of a bonus out of the profit derived from the sale. Thus, the payment of a bonus to them out
of the gain
realized from the sale cannot be considered as a selling expense; nor can it be deemed
reasonable and necessary so as to make it deductible for tax purposes.
COLLECTOR OF INTERNAL REVENUE, petitioner, vs. GOODRICH
INTERNATIONAL RUBBER CO., respondent.
[G.R. No. L-22265. December 22, 1967.]
Facts: The CIR disallowed the bad debts and representation expense claimed as
deduction of the respondent for tax purposes. According to Goodrich the claim for
deduction of the representation expense is based upon receipts issued, not by the entities
in which the alleged expenses had been incurred, but by the officers of Goodrich who
allegedly paid them. To collect for the alleged bad debts, the respondent sent demand
letters. There were subsequent collections after the debts have been written.
Issue: WON the representation expense are valid deductions.
Held: No. If the expenses had really been incurred, receipts or chits would have been
issued by the entities to which the payments had been made, and it would have been easy
for Goodrich or its officers to produce such receipts. Those issued by said officers merely
attest to their claim that they had incurred and paid said expenses. They do not establish
payment of said alleged expenses to the entities in which the same are said to have been
incurred.
Issue: WON the bad debts are valid deduction for income tax purposes
Held: No. The ascertainment of worthlessness of bad debts requires proof of two facts: (1)
that the taxpayer did in fact ascertain the debt to be worthless in the year the deduction is
sought; and (2) in so doing, he acted in good faith. Good faith is not enough. The taxpayer
must show
that he had reasonably investigated the relevant facts and had drawn a reasonable
inference from the information thus obtained by him. In this case, there were payments
made after it has been written off and proves that there is undue haste in claiming it as bad
debts. Respondent has not proven that said debts were worthless. There is no evidence
that thedebtors cannot pay them.
COLLECTOR OF INTERNAL REVENUE, petitioner, vs. ALBERTO M.
K. JAMIR, respondent.
[G.R. No. L-16552. March 30, 1962.]
Facts: The CIR assessed the respondent for deficiency income taxes. The Petitioner
claimed that the respondent under declared its income based on the expenditure method.
The petitioner considered as an undeclared income so much of respondents expenditures
for said months as was in excess of his reported income for the same months
Issue: WON the expenditure method was properly applied.
Held: No. The "expenditures method" of determining income should be applied by
deducting the aggregate yearly expenditures from the declared yearly income, not the
expenditures incurred each month from the declared income therefor. In this case the
respondent properly explained that the income derived from the advances from customers
were entered in his books of account in subsequent months.
Issue: WON the deduction for car depreciation and drivers expense is proper.

Held: Yes. In this case, the car was used by Jamir for both personal and business
purposes, the lower court allowed, as deductions, three-fourths (3/4) of said amounts, the
car having been used by Jamir "more for business than for personal purposes".
HOSPITAL DE SAN JUAN DE DIOS, INC. vs. COMMISSIONER OF
INTERNAL REVENUE
Facts: Petitioner is engaged in both taxable and non-taxable operations. For the years
1952 to 1955, the petitioner allocated its administrative expenses. The respondent
disallowed, however, the interests and dividends from sharing in the allocation of
administrative expenses on the ground that the expenses incurred in the administration or
management of petitioner's investments are not allowable business expenses inasmuch as
they were not incurred in 'carrying on any trade or business' within the contemplation of
Section 30 (a)(1) of the Revenue
Code. Hence, were assessed for deficiency income taxes.
Issue: WON administrative expenses should be considered as a deduction/allocated to its
interest and dividend income for income tax purposes.
Held: No. the principle of allocating expenses is grounded on the premise that the taxable
income was derived from carrying on a trade or business, as distinguished from mere
receipt of interests and dividends from one's investments, the Court of Tax Appeals
correctly ruled that
said income should not share in the allocation of administrative expenses. Hospital de San
Juan De Dios, Inc., according to its Articles of Incorporation, was established for purposes
"which are benevolent, charitable and religious, and not for financial gain". It is not carrying
on a trade or business for the word "business" in its ordinary and common use means
"human efforts which have for their end living or reward; it is not commonly used as
descriptive of charitable, religious, educational or social agencies" or "any particular
occupation or employment habitually engaged in especially for livelihood or gain" or
"activities where profit is the purpose or livelihood is the motive."
FELIX MONTENEGRO, INC., petitioner, vs. COMMISSIONER OF
INTERNAL REVENUE, respondent.
[C.T.A. CASE NO. 695. April 30, 1969.]
Facts: The CIR disallowed salaries of some of its officers, value of medicines, campaign
contribution and miscellaneous expense as deduction for income tax purposes hence the
petitioner was assessed for deficiency income taxes. The deduction of salaires was
disallowed
because the said officers are also stockholders of the corporation and that their salaries are
excessive compared to those of officers of other corporation holding similar positions and
doing the same volume of business.
Issue: WON the salaries and expenses should be allowed as a deduction.
Held: Yes. The general rule is that the employer is given a wide latitude of discretion in the
amount of salaries paid to the employees. A corporation has the right to fix the
compensation of its employees .There is no comparative study of the profits of the two
enterprises in relation to
other concerns similarly situated. Neither is there any comparative study of the peculiar
situation of the two enterprises in relation to other concerns, nor is there a comparison of
the nature and volume of the work performed by the officers involved. Since no two
business enterprises are exactly in the same situation, negligible differences in salaries
cannot reasonably show that the salary is excessive or that profits are channeled to the
stockholders thru salaries.
Issue: WON the value of medicines is a deductible loss
Held: No. Aside from self serving testimonial evidence, no other evidence was presented to
substantiate this claim of petitioner. There is not even a list of the medicines, their value,
and their expiry dates. The deductible as loss on the ground that the aforesaid medicines
were no
longer fit for sale as the dates of their efficacy have expired should be disallowed.
Issue: WON campaign contribution should be allowed as a deduction.

Held: No. Amount expended for political campaign purposes or payments to campaign
funds are not deductible either as business expenses or as contribution
COLLECTOR V. PHILIPPINE EDUCATION CO.
FACTS: Respondent lost all its pre-war books of accounts and records, with the exception
of a copy of the trial balance sheet. It employed an accounting firm and paid it the sum of P
13, 045.48. to prepare and prove its war damage claim. In filing its income tax return
respondent
claimed said sum as deduction under section 30 of the NIRC. Petitioner disallowed the
same and instead assessed additional P2,405.14 as deficiency income tax. CTA reversed
upon appeal and declared respondent exempt from the deficiency income tax in question.
ISSUE: Whether or not the expense in question was ordinary and necessary and whether
or not it was paid or incurred in carrying on respondents business.
HELD: Yes. The law does not say that the expense must be for or on account of
transactions in ones trade or business. Ordinarily an expense will be considered necessary
where the expenditure is appropriate and helpful in the development of the taxpayers
business. It is sufficient that the expense were incurred for purposes proper to the conduct
of the corporate affairs or for the purpose of realizing a profit or of minimizing a loss. The
fee in question was paid by the respondent to recover its lost assets occasioned by the war
and thereby to be so rehabilitated as to be able to carry on its business. Also, it should be
noted that even if there is no law exempting the proceeds of war damage claims from
taxes, the war damage compensation would still not be subject to tax, not being an income.
Compensation for injury to capital is never income.
DOCTRINE: To carry on its business the taxpayer not only must have sufficient assets but
must preserve the same and recover any that should be lost. The fee or expense paid to
recover its lost assets occasioned by the war and thereby to be so rehabilitated as to be
able to carry on its business is not required that it must be for or on account of transactions
in ones trade or business.
DEDUCTIONS AND EXEMPTIONS; ALLOWABLE DEDUCTIONS;
INTEREST EXPENSE
CIR v. PALANCA
FACTS: Don Carlos Palanca, Sr. donated in favor of his son, the petitioner, herein shares
of stock in La Tondea, Inc. amounting to 12,500 shares. For failure to file a return on the
donation within the statutory period, the petitioner was assessed the sums of P97,691.23,
P24,442.81 and P47,868.70 as gift tax, 25% surcharge and interest, respectively, which he
paid on June 22, 1955. The petitioner filed with the BIR his income tax return for the
calendar year 1955, claiming, among others, a deduction for interest amounting to
P9,706.45 and reporting a taxable income of P65,982.12. On the basis of this return, he
was assessed the sum of P21,052.91, as income tax, which he paid, as follows: Petitioner
filed an amended return for the calendar year 1955, claiming therein an additional
deduction in the amount of P47,868.70 representing interest paid on the donee's gift tax,
thereby reporting a taxable net income of P18,113.42 and a tax due thereon in the sum of
P3,167.00. The claim for deduction was based on the provisions of Section 30(b) (1) of the
Tax Code, which authorizes the deduction from gross income of interest paid within the
taxable year on indebtedness. A claim for the refund of alleged overpaid income taxes for
the year 1955 amounting to P17,885.01, which is the difference between the amount of
P21,052.01 he paid as income taxes under his original return and of P3,167.00, was filed
together with this amended return. BIR denied the claim. On August 12, 1958, the petitioner
once more filed an amended income tax return for the calendar year 1955, claiming, in
addition to the interest deduction of P9,076.45 appearing in his original return, a deduction
in the amount of P60,581.80, representing interest on the estate and inheritance taxes on
the 12,500 shares of stock, thereby reporting a net taxable income for 1955 in the amount
of P5,400.32 and an income tax due thereon in the sum of P428.00. Again this was denied.
CTA reversed.
ISSUE/S: 1) Whether the amount paid by respondent Palanca for interest on his delinquent
estate and inheritance tax is deductible from the gross income for that year under Section
30 (b) (1) of the Revenue Code; 2) Whether the claim for refund has prescribed.

HELD: 1) Yes. While "taxes" and "debts" are distinguishable legal concepts, in certain
cases as in the suit at bar, on account of their nature, the distinction becomes
inconsequential. We do not see any element in this case which can justify a departure from
or abandonment of the doctrine in the Prieto case. In both this and the said case, the
taxpayer sought the allowance as deductible items from the gross income of the amounts
paid by them as interests on delinquent tax liabilities. Of course, what was involved in the
cited case was the donor's tax while the present suit pertains to interest paid on the estate
and inheritance tax. This difference, however, submits no appreciable consequence to the
rationale of this Court's previous determination that
interests on taxes should be considered as interests on indebtedness within the meaning of
Section 30(b) (1) of the Tax Code. 2) No. The 30-day period under Section 11 of Republic
Act 1125 did not even commence to run in this incident. It should be recalled that while the
herein petitioner originally assessed the respondent-claimant for alleged gift tax liabilities,
the said assessment was subsequently abandoned and in its lieu, a new one was prepared
and served on the respondent-taxpayer. In this new assessment, the petitioner charged the
said respondent with an entirely new liability and for a substantially different amount from
the first. While initially the petitioner assessed the respondent for donee's gift tax in the
amount of P170,002.74, in the
subsequent assessment the latter was asked to pay P191,591.62 for delinquent estate and
inheritance tax. Considering that it is the interest paid on this latter-assessed estate and
inheritance tax that respondent Palanca is claiming refund for, then the thirty-day period
under the abovementioned section of Republic Act 1125 should be computed from the
receipt of the final denial by the Bureau of Internal Revenue of the said claim. In the second
place, the claim at bar refers to the alleged overpayment by respondent Palanca of his
1955 income tax. Inasmuch as the said account was paid by him by installment, then the
computation of the two year prescriptive period, under Section 306 of the National Internal
Revenue Code, should be from the date of the last installment.
DOCTRINE: While "taxes" and "debts" are distinguishable legal concepts, in certain cases
as in the suit at bar, on account of their nature, the distinction becomes inconsequential.
CIR V VIUDA DE PRIETO
FACTS: Respondent conveyed by way of gifts to her four children, namely, Antonio, Benito,
Carmen and Mauro, all surnamed Prieto, real property with a total assessed value of
P892,497.50. After the filing of the gift tax returns on or about February 1, 1954, the
petitioner CIR appraised the real property donated for gift tax purposes at P1,231,268.00,
and assessed the total sum of P117,706.50 as donor's gift tax, interest and compromises
due thereon. Of the total sum of P117,706.50 paid by respondent on April 29, 1954, the
sum of P55,978.65 represents the total interest on account of deliquency. This sum of
P55,978.65 was claimed as deduction, among others, by respondent in her 1954 income
tax return. Petitioner, however,
disallowed the claim and as a consequence of such disallowance assessed respondent for
1954 the total sum of P21,410.38 as deficiency income tax due on the aforesaid
P55,978.65, including interest up to March 31, 1957, surcharge and compromise for the
late payment. Under the law, for interest to be deductible, it must be shown that there be an
indebtedness, that there should be interest upon it, and that what is claimed as an interest
deduction should have been paid or accrued within the year. It is here conceded that the
interest paid by respondent was in consequence of the late payment of her donor's tax, and
the same was paid within the year it is sought to be declared.
ISSUE/S: Whether or not such interest was paid upon an indebtedness within the
contemplation of section 30 (b) (1) of the Tax Code.
HELD: Yes. The term "indebtedness" as used in the Tax Code of the United States
containing similar provisions as in the above-quoted section has been defined as an
unconditional and legally enforceable obligation for the payment of money. Although taxes
already due have not, strictly speaking, the same concept as debts, they are, however,
obligations that may be considered as such. The term "debt" is properly used in a
comprehensive sense as embracing not merely money due by contract but whatever one is
bound to render to another, either for contract, or the requirement of the law. It follows that
the interest paid by herein respondent for the late payment of her donor's tax is deductible

from her gross income under section 30(b) of the Tax Code above quoted. This conclusion
finds support in the established jurisprudence in the United States after whose laws our
Income Tax Law has been patterned. Thus, under sec. 23(b) of the Internal Revenue Code
of 1939, as amended , which contains similarly
worded provisions as sec. 30(b) of our Tax Code, the uniform ruling is that interest on taxes
is interest on indebtedness and is deductible.
DOCTRINE: The term "indebtedness" as used in the Tax Code of the United States
containing similar provisions as in the abovequoted section has been defined as an
unconditional and legally enforceable obligation for the payment of money.
PAPER INDUSTRIES V CA
FACTS: Petitioner is registered with the BOI as a preferred pioneer enterprise with respect
to its integrated pulp and paper mill, and as a preferred non-pioneer enterprise with respect
to its integrated plywood and veneer mills. It received from the CIR two (2) letters of
assessment
and demand (a) one for deficiency transaction tax and for documentary and science stamp
tax; and (b) the other for deficiency income tax for 1977, for an aggregate amount of
P88,763,255.00. Picop protested the assessment of deficiency transaction tax and
documentary and science stamp taxes. These protests were not formally acted upon by
respondent CIR. On 26 September 1984, the CIR issued a warrant of distraint on personal
property and a warrant of levy on real property against Picop, to enforce collection of the
contested assessments; in effect, the CIR denied Picop's protests. Thereupon, Picop went
before the CTA. Picop and the CIR both went to the Supreme Court on separate Petitions
for Review of the above decision of the CTA. In two (2) Resolutions dated 7 February 1990
and 19 February 1990, respectively, the Court referred the two (2) Petitions to the Court of
Appeals. The Court of Appeals consolidated the two (2) cases and rendered a decision,
dated 31 August 1992, which further reduced the liability of Picop to P6,338,354.70. Picop
now maintains that it is not liable at all to pay any of the
assessments or any part thereof. It assails the propriety of the thirty-five percent (35%)
deficiency transaction tax which the Court of Appeals held due from it in the amount of
P3,578,543.51. Picop also questions the imposition by the Court of Appeals of the
deficiency income tax of P1,481,579.15, resulting from disallowance of certain claimed
financial guarantee expenses and claimed year-end adjustments of sales and cost of sales
figures by Picop's external auditors. 3 The CIR, upon the other hand, insists that the Court
of Appeals erred in
finding Picop not liable for surcharge and interest on unpaid transaction tax and for
documentary and science stamp taxes and in allowing Picop to claim as deductible
expenses.
ISSUE/S: 1) Whether Picop is liable for the thirty-five percent (35%) transaction tax; 2)
Whether Picop is liable for interest and surcharge on unpaid transaction tax; 3) Whether
Picop is entitled to deduct against current income interest payments on loans for the
purchase of machinery and equipment; 4) Whether Picop is entitled to deduct against
current income net operating losses incurred by Rustan Pulp and Paper Mills, Inc; 5)
Whether Picop is entitled to deduct against current income certain claimed financial
guarantee expenses; 6) Whether Picop had understated its sales and overstated its cost of
sales for 1977; 7) Whether Picop is liable for the corporate development tax of five percent
(5%) of its income for 1977.
HELD: 1) We agree with the CTA and the Court of Appeals that Picop's tax exemption
under R.A. No. 5186, as amended, does not include exemption from the thirty-five percent
(35%) transaction tax. In the first place, the thirty-five percent (35%) transaction tax is an
income tax, that is, it is a tax on the interest income of the lenders or creditors. It is thus
clear that the transaction tax is an income tax and as such, in any event, falls outside the
scope of the tax exemption granted to registered pioneer enterprises by Section 8 of R.A.
No. 5186, as amended. 2) Section 51 (c) and (e) of the 1977 Tax Code did not authorize
the imposition of a surcharge and penalty interest for failure to pay the thirtyfive percent
(35%) transaction tax imposed under Section 210 (b) of the same Code. The corresponding
provision in the current Tax Code very

clearly embraces failure to pay all taxes imposed in the Tax Code, without any regard to the
Title of the Code where provisions imposing particular taxes are textually located. Tax
exemptions are, to be sure, to be "strictly construed," that is, they are not to be extended
beyond the ordinary and reasonable intendment of the language actually used by the
legislative authority in granting the exemption. The issuance of debenture bonds is certainly
conceptually distinct from pulping and paper manufacturing operations. But no one
contends that issuance of bonds was a principal or regular business activity of Picop; only
banks or other financial institutions are in the
regular business of raising money by issuing bonds or other instruments to the general
public.
3) We have already noted that our 1977 NIRC does not prohibit the deduction of interest on
a loan incurred for acquiring machinery and equipment. Neither does our 1977 NIRC
compel the capitalization of interest payments on such a loan. The 1977 Tax Code is simply
silent on
a taxpayer's right to elect one or the other tax treatment of such interest payments.
Accordingly, the general rule that interest payments on a legally demandable loan are
deductible from gross income must be applied. We conclude that the CTA and the Court of
Appeals did not err in allowing the deductions of Picop's 1977 interest payments on its
loans for capital equipment against its gross income for 1977. 4) After prolonged
consideration and analysis of this matter, the Court is unable to agree with the CTA and
Court of Appeals on the deductibility of RPPM's accumulated losses against Picop's 1977
gross income. It is important to note at the outset that in our jurisdiction, the ordinary rule
that is, the rule applicable in respect of corporations not
registered with the BOI as a preferred pioneer enterprise is that net operating losses
cannot be carried over. Under our Tax Code, both in 1977 and at present, losses may be
deducted from gross income only if such losses were actually sustained in the same year
that they are deducted or charged off. Thus it is that R.A. No. 5186 introduced the carryover of net operating
losses as a very special incentive to be granted only to registered pioneer enterprises and
only with respect to their registered operations. In the instant case, to allow the deduction
claimed by Picop would be to permit one corporation or enterprise, Picop, to benefit from
the operating
losses accumulated by another corporation or enterprise, RPPM. In effect, to grant Picop's
claimed deduction would be to permit Picop to purchase a tax deduction and RPPM to
peddle its accumulated operating losses. We consider and so hold that there is nothing in
Section 7 (c) of
R.A. No. 5186 which either requires or permits such a result. Indeed, that result makes
non-sense of the legislative purpose which may be seen clearly to be projected by Section
7 (c), R.A. No. 5186. We conclude that the deduction claimed by Picop in the amount of
P44,196,106.00 in its 1977 Income Tax Return must be disallowed. 5) We must support the
CTA and the Court of Appeals in their foregoing rulings. A taxpayer has the burden of
proving entitlement to a claimed
deduction. Even Picop's own vouchers were not submitted in evidence and the BIR
Examiners denied that such vouchers and other documents had been exhibited to them.
Moreover, cash vouchers can only confirm the fact of disbursement but not necessarily the
purpose thereof.
6) The CIR has made out at least a prima facie case that Picop had understated its sales
and overstated its cost of sales as set out in its Income Tax Return. For the CIR has a right
to assume that Picop's Books of Accounts speak the truth in this case since, as already
noted,
they embody what must appear to be admissions against Picop's own interest.
7) The adjusted net income of Picop for 1977, as will be seen below, is P48,687,355.00. Its
net worth figure or total stockholders' equity as reflected in its Audited Financial Statements
for 1977 is P464,749,528.00. Since its adjusted net income for 1977 thus exceeded ten
percent (10%) of its net worth, Picop must be held liable for the five percent (5%) corporate
development tax in the amount of P2,434,367.75.
DOCTRINE: It is thus clear that the transaction tax is an income tax and
as such, in any event, falls outside the scope of the tax exemption

granted to registered pioneer enterprises by Section 8 of R.A. No. 5186,


as amended.
CIR v. ITOGON-SUYOC MINES, INC.
FACTS: Respondent Itogon-Suyoc Mines, Inc., the taxpayer involved, duly paid in full its
liability according to its income tax return for the fiscal year 1960-61. Instead, it deducted
right away the amount represented by claim for refund filed eight (8) months back, for the
previous year's income tax, for which it was not liable at all, so it alleged, as it suffered a
loss instead, a claim subsequently favorably acted on by petitioner Commissioner of
Internal Revenue but after the date of such payment of the 1960-1961 tax. Accordingly, an
interest in the amount of P1,512.83
was charged by petitioner Commissioner of Internal Revenue on the sum withheld on the
ground that no deduction on such refund should be allowed before its approval. When the
matter was taken up before the Court of Tax Appeals, the above assessment representing
interest was
set aside in the decision of September 30, 1965.
ISSUE/S: Whether CTA should not have absolved respondent corporation "from liability to
pay the sum of P1,512.83 as 1% monthly interest for delinquency in the payment of income
tax for the fiscal year 1960-1961."
HELD: No. It could not be error for the Court of Tax Appeals, considering the admitted fact
of overpayment, entitling respondent to refund, to hold that petitioner should not repose an
interest on the aforesaid sum of P13,155.20 "which after all was paid to and received by
the government
even before the incidence of the tax in question." It would be, according to the Court of Tax
Appeals, "unfair and unjust" to do so. We agree but we go farther. The imposition of such
an interest by petitioner is not supported by law.
The National Internal Revenue Code provides that interest upon the amount determined as
a deficiency shall be assessed and shall be paid upon notice and demand from the
Commissioner of Internal Revenue. There is no question respondent was entitled to a
refund. Instead of waiting for the sum involved to be delivered to it, it deducted the said
amount from the tax that it had to pay. That it had a right to do according to the law. It is
true a doubt could have arisen due to the fact that as of the time such a deduction was
made, the Commissioner of Internal Revenue had not as yet approved such a refund. It is
an admitted fact though that respondent was clearly entitled to it, and petitioner did not
allege otherwise. Nor could he do so. Under all the circumstances disclosed therefore, the
applicability of the legal provision allowing such a deduction from the amount of the tax to
be paid cannot be disputed.
DOCTRINE: It could not be error for the Court of Tax Appeals, considering the admitted
fact of overpayment, entitling respondent to refund, to hold that petitioner should not
repose an interest on the aforesaid sum of P13,155.20 "which after all was paid to and
received by the government even before the incidence of the tax in question."
CASTRO v. COLLECTOR
Facts: This is an appeal from a decision of the Court of Tax Appeals (in its C.T.A. Case
141) holding petitioner Maria B. Castro liable under the War Profits Tax Law, Republic Act
No. 55, and ordering her to pay a deficiency war profits tax (including surcharges and
interest) in the
amount of P1,360,514.66, and costs. Castro was previously acquitted in the criminal case
instituted against her for violation of the War Profits Tax Law.
Issue: WON the acquittal is a bar to the collection of the taxes assessed, and specially of
the 50% surcharge
Held: NO. With regard to the tax proper, the state correctly points out in its brief that the
acquittal in the criminal case could not operate to discharge petitioner from the duty to pay
the tax, since that duty is imposed by statute prior to and independently of any attempts on
the part of the taxpayer to evade payment. The obligation to pay the tax is not a mere
consequence of the felonious acts charged in the information, nor is it a mere civil liability
derived from crime that would be wiped out by the judicial declaration that the criminal acts
charged did not exist. As to the 50% surcharge, the very United States Supreme Court that

rendered the Coffey decision has subsequently pointed out that additions of this kind to the
main tax are not penalties but civil administrative sanctions, provided primarily as a
safeguard for the protection of the state revenue and to reimburse the government for the
heavy expense of investigation and the loss resulting from the taxpayer's fraud (Helvering
vs. Mitchell, 303 U.S. 390, 82 L. Ed. 917; Spies vs. U.S. 317 U.S. 492). This is made plain
by the fact that such surcharges are enforceable, like the primary tax itself, by distraint or
civil suit, and that they are provided in a section of R.A. No. 55 (section 5) that is separate
and distinct from that providing for criminal prosecution (section 7). We conclude that the
defense of jeopardy and estoppel by reason of the petitioner's acquittal is
untenable and without merit. Whether or not there was fraud committed by the taxpayer
justifying the imposition of the surcharge is an issue of fact to be inferred from the evidence
and surrounding circumstances; and the finding of its existence by the Tax Court is
conclusive upon us. (Gutierrez v. Collector, G.R. No. L-9771, May 31, 1951 ; Perez vs.
Collector, supra).
COLLECTOR v. FISHER
Facts: This case relates to the determination and settlement of the hereditary estate left by
the deceased Walter G. Stevenson, and the laws applicable thereto. Walter G. Stevenson
(born in the Philippines on August 9, 1874 of British parents and married in the City of
Manila on January 23, 1909 to Beatrice Mauricia Stevenson another British subject) died
on February 22, 1951 in San Francisco, California, U.S.A. whereto he and his wife moved
and established their permanent residence since May 10, 1945. In his will executed in San
Francisco on May 22, 1947, and which was duly probated in the Superior Court of
California on April 11, 1951, Stevenson instituted his wife Beatrice as his sole heiress to the
following real and personal properties acquired by the spouses while residing in the
Philippines.
Issue: 1. Whether or not the estate is entitled to the following deductions: P8,604.39 for
judicial and administration expenses; P2,086.52 for funeral expenses; P652.50 for real
estate taxes; and
P10,0,22.47 representing the amount of indebtedness allegedly incurred by the decedent
during his lifetime
2. Whether or not the estate is entitled to the payment of interest on the amount it claims to
have overpaid the government and to be refundable to it.
Held: 1. YES. An examination of the record discloses, however, that the foregoing items
were considered deductible by the Tax Court on the basis of their approval by the probate
court to which said expenses, we may presume, had also been presented for
consideration. It is to be
supposed that the probate court would not have approved said items were they not
supported by evidence presented by the estate. In allowing the items in question, the Tax
Court had before it the pertinent order of the probate court which was submitted in
evidence by respondents. (Exh. "AA-2", p. 100, record). As the Tax Court said, it found no
basis for departing from the findings of the probate court, as it must have been satisfied
that those expenses were actually incurred. Under the circumstances, we see no ground to
reverse this finding of fact which, under Republic Act of California National Association,
which it would appear, that while still living, Walter G. Stevenson obtained we are not
inclined to pass upon the claim of respondents in respect to the
additional amount of P86.52 for funeral expenses which was disapproved by the court a
quo for lack of evidence. In connection with the deduction of P652.50 representing the
amount of realty taxes paid in 1951 on the decedent's two parcels of land in Baguio City,
which respondents claim was disallowed by the Tax Court, we find that this claim has in fact
been allowed.
2. NO. Respondent's claim for interest on the amount allegedly overpaid, if any actually
results after a recomputation on the basis of this decision is hereby denied in line with our
recent decision in Collector of Internal Revenue v. St. Paul's Hospital (G.R. No. L-12127,
May 29, 1959) wherein we held that, "in the absence of a statutory provision clearly or
expressly directing or authorizing such payment, and none has been cited by respondents,
the National Government cannot be required to pay interest."

DEDUCTIONS AND EXMEPTIONS; ALLOWABLE DEDUCTIONS;


TAXES
CIR v. LEDNICKY
Facts: V. E. Lednicky and Maria Valero Lednicky, are husband and wife, both American
citizens residing in the Philippines, and have derived all their income from Philippine
sources for the taxable years under question. [GR L-18286] In compliance with local law,
the spouses, on 27
March 1957, filed their income tax return for 1956, reporting therein a gross income of
P1,017,287.65 and a net income of P733,809.44 on which the amount of P317,395.41 was
assessed after deducting P4,805.59 as withholding tax. Pursuant to the Commissioner of
Internal Revenues assessment notice, the spouses paid the total amount of P326,247.41,
inclusive of the withheld taxes, on 15 April 1957. On 17 March 1959, the spouses filed an
amended income tax return for 1956. The amendment consists in a claimed deduction of
P205,939.24 paid in 1956 to the US government as federal income tax for 1956.
Simultaneously with the filing of the amended return, the spouses requested the refund of
P112,437.90. When the Commissioner of Internal Revenue failed to answer the claim for
refund, the spouses filed
their petition with the tax court on 11 April 1959 as CTA Case 646. [GR L-18165] On 28
February 1956, the spouses filed their domestic income tax return for 1955, reporting a
gross income of P1,771,124.63 and a net income of P1,052,550.67. On 19 April 1956, they
filed an amended income tax return, the amendment upon the original being a lesser net
income of P1,012,554.51, and, on the basis of this amended return, they paid P570,252.00,
inclusive of withholding taxes. After audit, the Commissioner determined a deficiency of
P16,116.00, which amount the spouses paid on 5 December 1956. Back in 1955, however,
the spouses filed with the US Internal Revenue Agent in Manila their Federal income tax
return for the years 1947, 1951, 1952, 1953 and 1954 on income from Philippine sources
on a cash basis. Payment of these federal income taxes, including penalties and
delinquency interest in the amount of $264,588.82, were made in 1955 to the US Director
of Internal Revenue, Baltimore, Maryland, through the National City Bank of New York,
Manila Branch. Exchange and bank charges in remitting payment totaled P4,143.91. On 11
August 1958 the said respondents amended their Philippines income tax return for 1955 to
including US Federal income taxes, interest accruing up to 15 May 1955, and exchange
and bank charges, totaling P516,345.15 and therewith filed a claim for refund of the sum of
P166,384.00, which was later reduced to P150,269.00. The spouses brought suit in the Tax
Court, which was docketed therein as CTA Case 570. [GR 21434] The facts are similar to
above cases but refer to the spouses income tax returns for 1957, filed on 28 February
1958, and for which the spouses paid a total sum of P196,799.65. In 1959, they filed an
amended return for 1957, claiming deduction of P190,755.80, representing taxes paid to
the US Government on income derived wholly from Philippine sources. On the strength
thereof, spouses seek refund of P90,520.75 as overpayment (CTA Case 783). The Tax
Court decided for the spouses.
Issue: WON there should be a refund for the spouses
Held: NO. The Supreme Court reversed the decisions of the Court of Tax Appeals, and
affirmed the disallowance of the refunds claimed by the spouses, with costs against said
spouses.
1. Section 30 (c-1) of the Philippine Internal Revenue Code Section 30 (c) (1)
(Deduction from gross income) provides that in computing net income there shall be
allowed as deductions: (c) Taxes: (1) In general. Taxes paid or accrued within the
taxable year, except (A) The income tax provided for under this Title; (B) Income, warprofits, and excess profits taxes imposed by the authority of any foreign country; but this
deduction shall be allowed in the case of a taxpayer who does not signify in his return his
desire to have to any extent the benefits of
paragraph (3) of this subsection (relating to credit for taxes of foreign countries); (C) Estate,
inheritance and gift taxes; and (D) Taxes assessed against local benefits of a kind tending
to increase the value of the property assessed.
2. Paragraph (c) (3) (b) of the Tax Code; Credits against tax for taxes of foreign
countries

Paragraph 3 (B) of the subsection (Credits against tax for taxes of foreign countries), reads:
If the taxpayer signifies in his return his desire to have the benefits of this paragraph, the
tax imposed by this Title shall be credited with (B) Alien resident of the Philippines. In the
case of an alien resident of the Philippines, the amount of any such taxes paid or accrued
during the taxable year to any foreign country, if the foreign country of which such alien
resident is a citizen or subject, in imposing such taxes, allows a similar credit to citizens of
the Philippines residing in
such country;
3. Paragraph (c) (4) of the Tax Code; Limitation on credit The tax credit so authorized is
limited under paragraph 4 (A and B) of the same subsection, in the following terms: Par. (c)
(4) Limitation on credit. The amount of the credit taken under this section shall be
subject to each of the following limitations: (A) The amount of the credit in respect to the tax
paid or accrued to any country shall not exceed the same proportion of the tax against
which such credit is taken, which the taxpayers net income from sources within such
country taxable under this Title bears to his entire net income for the same taxable year;
and (B) The total amount of the credit shall not exceed the same proportion of the tax
against which such credit is taken, which the taxpayers net income from sources without
the Philippines taxable under this Title bears to his entire net income for the same taxable
year.
4. Laws intent that right to deduct income taxes paid to foreign government taken as
an alternative or substitute to claim of tax credit for such foreign income tax
Construction and wording of Section 30 (c) (1) (B) of the Internal Revenue Act shows the
laws intent that the right to deduct income taxes paid to foreign government from the
taxpayers gross income is given
only as an alternative or substitute to his right to claim a tax credit for such foreign income
taxes under section 30 (c) (3) and (4); so that unless the alien resident has a right to claim
such tax credit if he so chooses, he is precluded from deducting the foreign income taxes
from his gross income. For it is obvious that in prescribing that such deduction shall be
allowed in the case of a taxpayer who does not signify in his return his desire to have to
any extent the benefits of paragraph (3) (relating to credits for taxes paid to foreign
countries), the statute assumes that the taxpayer in question also may signify his desire, to
claim a tax credit and waive the deduction; otherwise, the foreign taxes would always be
deductible, and their mention in the list of nondeductible items in Section 30 (c) might as
well have been omitted, or at least expressly limited to taxes on income from sources
outside the Philippine Islands. Had the law intended that foreign income taxes could be
deducted from gross income in any event, regardless of the
taxpayers right to claim a tax credit, it is the latter right that should be conditioned upon the
taxpayers waiving the deduction; in which case the right to reduction under subsection (c1-B) would have been made absolute or unconditional (by omitting foreign taxes from the
enumeration of non- deductions), while the right to a tax credit under subsection (c-3)
would have been expressly conditioned upon the taxpayers not claiming any deduction
under subsection (c-1).
5. Danger of double credit does not exist if taxpayer cannot claim benefit from either
headings at his option The purpose of the law is to prevent the taxpayer from claiming
twice the benefits of his payment of foreign taxes, by deduction from gross income (subs. c1) and by tax credit (subs. c-3). This danger of double credit certainly can not exist if the
taxpayer can not claim benefit under either of these headings at his option, so that he must
be entitled to a tax credit (the spouses admittedly are not so entitled because all their
income is derived from Philippine sources), or the option to deduct from gross income
disappears altogether.
6. When double taxation; Tax income should accrue to benefit of the Philippines
Double taxation becomes obnoxious only where the taxpayer is taxed twice for the benefit
of the same governmental entity (cf. Manila vs. Interisland Gas Service, 52 Off. Gaz. 6579,
Manuf. Life Ins. Co. vs. Meer, 89 Phil. 357). In the present case, while the taxpayers would
have to pay two taxes on the same income, the Philippine government only receives the
proceeds of one tax. As between the Philippines, where the income was earned and where
the taxpayer is domiciled, and the United States, where that income was not earned and
where the taxpayer did not reside, it is indisputable that justice and equity demand that the

tax on the income should accrue to the benefit of the Philippines. Any relief from the alleged
double taxation should come from the United States, and not from the Philippines, since the
formers right to burden the taxpayer is solely predicated on his citizenship, without
contributing to the production of the wealth that is being taxed. To allow an alien resident to
deduct from his gross income whatever taxes he pays to his own government amounts to
conferring on the latterpower to reduce the tax income of the Philippine government simply
by increasing the tax rates on the alien resident. Everytime the rate of taxation imposed
upon an alien resident is increased by his own government, his deduction from Philippine
taxes would correspondingly increase, and the proceeds for the Philippines diminished,
thereby subordinating our own taxes to those levied by a foreign government. Such a result
is incompatible with the status of the Philippines as an independent and sovereign state.
GUTIERREZ v. COLLECTOR
Facts: Maria Morales was the registered owner of an agricultural land designated as Lot
No. 724-C of the cadastral survey of Mabalacat, Pampanga. The Republic of the
Philippines, at the request of the U.S.Government and pursuant to the terms of the Military
Bases Agreement of March 14, 1947, instituted condemnation proceedings in the Court of
the First Instance of Pampanga, docketed, as Civil Case No. 148, for the purpose of
expropriating the lands owned by Maria Morales and others needed for the expansion of
the Clark Field Air Base. t the commencement of the action, the Republic of the Philippines,
therein plaintiff deposited with the Clerk of the Court of First Instance of Pampanga the sum
of P156,960, which was provisionally fixed as the value of the lands sought to be
expropriated, in order that it could take immediate possession of the same. On January 27,
1949, upon order of the Court, the sum of P34,580 (PNB Check 721520-Exh. R) was paid
by the Provincial treasurer of Pampanga to Maria Morales out of the original deposit of
P156,960 made by therein plaintiff. After due hearing, the Court of First Instance of
Pampanga rendered decision dated November 29, 1949, wherein it fixed as just
compensation P2,500 per hectare for some of the lots and P3,000 per hectare for the
others, which values were based on the reports of the Commission on Appraisal whose
members were chosen by both parties and by the Court, which took into consideration the
different conditions affecting, the value of the condemned properties in making their
findings.
In virtue of said decision, defendant Maria Morales was to receive the amount of
P94,305.75 as compensation for Lot No. 724-C which was one of the expropriated lands.
Sometime in 1950, the spouses Blas Gutierrez and Maria Morales received the sum of
P59.785.75 presenting the balance remaining in their favor after deducting the amount of
P34,580 already withdrawn from the compensation to them. In a notice of assessment
dated January 28, 1953, the Collector of
Internal Revenue demanded of the petitioners the payment of P8,481 as alleged deficiency
income tax for the year 1950, inclusive of surcharges and penalties. The CIR contended
that petitioners-appellants failed to include from their gross income, in filing their income tax
return for 1950, the amount of P94,305.75 which they had received as compensation for
their land taken by the Government by expropriation proceedings. It is the contention of
respondent Collector of Internal Revenue that such transfer of property, for taxation
purposes, is "sale" and that the income derived therefrom is taxable. The lower court
exonerated petitioners from the 50 per cent surcharge
imposed on the latter, on the ground that the taxpayers' income tax return for 1950 is false
and/or fraudulent.
Issue: WON petitioners should pay surcharge Held: NO. It should be noted that the Court
of Tax Appeals found that the evidence did not warrant the imposition of said surcharge
because
the petitioners therein acted in good faith and without intent to defraud the Government.
The question of fraud is a question of fact which frequently requires a nicely balanced
judgement to answer. All the facts and circumstances surrounding the conduct of the tax
payer's business and all the facts incident to the preparation of the alleged fraudulent return
should be considered. (Mertens, Federal Income Taxation, Chapter 55). The question of
fraud being a question of fact and the lower court having made the finding that "the
evidence of this case does not warrant the imposition of the 50 per cent surcharge", We are

constrained to refrain from giving any consideration to the question raised by the Solicitor
General, for it is already settled in this jurisdiction that in passing upon petitions to review
decisions of the Court of Tax Appeals, We have to confine ourselves to questions of law.
DEDUCTIONS AND EXEMPTIONS; ALLOWABLE DEDUCTIONS;
LOSSES
FERNANDEZ HERMANOS v. CIR
Facts: These four appeals involve two decisions of the Court of Tax Appeals determining
the taxpayer's income tax liability for the years 1950 to 1954 and for the year 1957. Both
the taxpayer and the Commissioner of Internal Revenue, as petitioner and respondent in
the cases a quo respectively, appealed from the Tax Court's decisions, insofar as their
respective contentions on particular tax items were therein resolved against them.
Issue: Proper/Improper Allowances/Disallowances of Losses
Held: Re allowances/disallowances of losses.
(a) Allowance of losses in Mati Lumber Co. (1950). The Commissioner of Internal
Revenue questions the Tax Court's allowance of the taxpayer's writing off as worthless
securities in its 1950 return the sum of P8,050.00 representing the cost of shares of stock
of Mati Lumber Co. acquired by the taxpayer on January 1, 1948, on the ground that the
worthlessness of said stock in the year 1950 had not been clearly established. The
Commissioner contends that although the said Company was no longer in operation in
1950, it still had its sawmill and equipment which must be of considerable value. There was
adequate basis for the writing off of the stock as worthless securities. Assuming that the
Company would later somehow realize some proceeds from its sawmill and equipment,
which were still existing as claimed by the Commissioner, and that such proceeds would
later be distributed to its stockholders such as the taxpayer, the amount so received by the
taxpayer would then properly be reportable as income of the taxpayer in the year it is
received.
(b) Disallowance of losses in or bad debts of Palawan Manganese Mines, Inc. (1951).
The taxpayer appeals from the Tax Court's disallowance of its writing off in 1951 as a loss
or bad debt the sum of P353,134.25, which it had advanced or loaned to Palawan
Manganese Mines, Inc. Pursuant to the agreement mentioned above, petitioner gave to
Palawan Manganese Mines, Inc. yearly advances starting from 1945, which advances
amounted to P587,308.07 by the end of 1951. Despite these advances and the resumption
of operations by Palawan Manganese
Mines, Inc., it continued to suffer losses. By 1951, petitioner became convinced that those
advances could no longer be recovered. While it continued to give advances, it decided to
write off as worthless the sum of P353,134.25. Under the circumstances, was the sum of
P353,134.25
properly claimed by petitioner as deduction in its income tax return for 1951, either as
losses or bad debts? It will be noted that in giving advances to Palawan Manganese Mine
Inc., petitioner did not expect to be repaid. It is true that some testimonial evidence was
presented to show that there was some agreement that the advances would be repaid, but
no documentary evidence was presented to this effect. The memorandum agreement
signed by the parties appears to be very clear that the consideration for the advances
made by petitioner was 15% of the net profits of Palawan Manganese Mines, Inc. In other
words, if there were no earnings or profits, there was no obligation to repay those
advances. It has been held that the voluntary advances made without expectation of
repayment do not result in deductible losses. The Tax Court's is allowance of the write-off
was proper. The Solicitor General has rightly pointed out that the taxpayer has taken an
"ambiguous position " and "has not definitely taken a stand on whether
the amount involved is claimed as losses or as bad debts but insists that it is either a loss
or a bad debt." 4 We sustain the government's position that the advances made by the
taxpayer to its 100% subsidiary, Palawan Manganese Mines, Inc. amounting to
P587,308,07 as of 1951 were
investments and not loans. 5 (c) Disallowance of losses in Balamban Coal Mines (1950
and
1951). The Court sustains the Tax Court's disallowance of the sums of P8,989.76 and
P27,732.66 spent by the taxpayer for the operation of its Balamban coal mines in Cebu in

1950 and 1951, respectively, and claimed as losses in the taxpayer's returns for said years.
The Tax Court correctly held that the losses "are deductible in 1952, when the mines were
abandoned, and not in 1950 and 1951, when they were still in operation." 9 The taxpayer's
claim that these expeditions should be allowed as losses for the corresponding years that
they were incurred,
because it made no sales of coal during said years, since the promised road or outlet
through which the coal could be transported from the mines to the provincial road was not
constructed, cannot be sustained. Some definite event must fix the time when the loss is
sustained, and here it was the event of actual abandonment of the mines in 1952. (d) and
(e) Allowance of losses in Hacienda Dalupiri (1950 to 1954) and Hacienda Samal (19511952). The Tax Court overruled the Commissioner's disallowance of these items of
losses thus: Petitioner deducted losses in the operation of its Hacienda Dalupiri the sums of
P17,418.95 in 1950, P29,125.82 in 1951, P26,744.81 in 1952, P21,932.62 in 1953, and
P42,938.56 in 1954. These deductions were disallowed by respondent on the ground that
the farm was operated solely for pleasure or as a
hobby and not for profit. This conclusion is based on the fact that the farm was operated
continuously at a loss. From the evidence, we are convinced that the Hacienda Dalupiri
was operated by petitioner for business and not pleasure. It was mainly a cattle farm,
although a few race horses were also raised. It does not appear that the farm was used by
petitioner for entertainment, social activities, or other nonbusiness purposes. Therefore, it is
entitled to deduct expenses and losses in connection with the operation of said farm. (See
1955 PH Fed. Taxes, Par. 13, 63, citing G.C.M. 21103, CB 1939- 1, p.164) Section 100 of
Revenue Regulations No. 2, otherwise known as the Income Tax Regulations, authorizes
farmers to determine their gross income on the basis of inventories. Said regulations
provide: "If gross income is ascertained by inventories, no deduction can be made for
livestock or products lost during the year, whether purchased for resale, produced on the
farm, as such losses will be reflected in the inventory by reducing the amount of livestock or
products on hand at the close of the year." Evidently, petitioner determined its income or
losses in the operation of said farm on the basis of inventories. We quote from the
memorandum of counsel for petitioner: "The Taxpayer deducted from its income tax returns
for the years from 1950 to 1954 inclusive, the corresponding yearly losses sustained in the
operation of Hacienda Dalupiri, which losses represent the excess of its yearly
expenditures over the receipts; that is, the losses represent the difference between the
sales of livestock and the actual cash disbursements or expenses." (Pages 21-22,
Memorandum for Petitioner.) As the Hacienda Dalupiri was operated by petitioner for
business and since it sustained losses in its operation, which losses were determined by
means of inventories authorized under Section 100 of Revenue Regulations No. 2, it was
error for respondent to have disallowed the deduction of said losses. The same is true with
respect to loss sustained in the operation of the Hacienda Samal for the years 1951 and
1952. 10 The Commissioner questions that the losses sustained by the taxpayer were
properly based on the inventory method of accounting. He concedes, however, "that the
regulations referred to does not specify how the inventories are to be made. The Tax Court,
however, felt satisfied with the evidence presented by the taxpayer ... which merely
consisted of an alleged physical count of the number of the livestock in Hacienda Dalupiri
for the years involved." 11 The Tax Court was satisfied with the method adopted by the
taxpayer as a farmer breeding livestock, reporting on the basis of receipts and
disbursements. We find no Compelling reason to disturb its findings.
PLARIDEL SURETY v. CIR
Facts: Petitioner Plaridel Surety & Insurance Co., is a domestic corporation engaged in the
bonding business. On November 9, 1950, petitioner, as surety, and Constancio San Jose,
as principal, solidarily executed a performance bond in the penal sum of P30,600.00 in
favor of the P. L. Galang Machinery Co., Inc., to secure the performance of San Jose's
contractual obligation to produce and supply logs to the latter. To afford itself adequate
protection against loss or damage on the performance bond, petitioner required San Jose
and one Ramon Cuervo to execute an indemnity agreement obligating themselves,
solidarily, to indemnify petitioner for whatever liability it may incur by reason of said
performance bond. Accordingly, San Jose constituted a chattel mortgage on logging

machineries and other movables in petitioner's favor1 while Ramon Cuervo executed a real
estate mortgage.2 San Jose later failed to deliver the logs to Galang Machinery3 and the
latter sued on the performance bond. On October 1, 1952, the Court of First Instance
adjudged San Jose and petitioner liable; it also directed San Jose and Cuervo to reimburse
petitioner for whatever amount it would pay Galang Machinery. The Court of Appeals, on
June 17, 1955, affirmed the judgment of the lower court. The same judgment was
likewise affirmed by this Court4 on January 11, 1957 except for a slight modification
apropos the award of attorney's fees. In its income tax return for the year 1957, petitioner
claimed the said
amount of P44,490.00 as deductible loss from its gross income and, accordingly, paid the
amount of P136.00 as its income tax for 1957. The Commissioner of Internal Revenue
disallowed the claimed deduction of P44,490.00 and assessed against petitioner the sum of
P8,898.00, plus interest, as deficiency income tax for the year 1957. Petitioner filed its
protest which was denied. Whereupon, appeal was taken to the Tax Court, petitioner
insisting that the P44,490.00 which it paid to Galang Machinery was a deductible loss.
Issue: WON the amount Plaridel paid to Galang Machinery is a deductible loss
Held: NO. There is no question that the year in which the petitioner Insurance Co. effected
payment to Galang Machinery pursuant to a final decision occurred in 1957. However,
under the same court decision, San Jose and Cuervo were obligated to reimburse
petitioner for whatever
payments it would make to Galang Machinery. Clearly, petitioner's loss is compensable
otherwise (than by insurance). It should follow, then, that the loss deduction can not be
claimed in 1957. Now, petitioner's submission is that its case is an exception. Citing Cu
Unjieng Sons, Inc. v. Board of Tax Appeals,6 and American cases also, petitioner argues
that even if there is a right to compensation by insurance or otherwise, the deduction can
be taken in the year of actual loss where the possibility of recovery is remote. The
pronouncement, however to this effect in the Cu Unjieng case is not as authoritative as
petitioner would have it since it was there found that the taxpayer had no legal right to
compensation either by insurance or otherwise.7 And the American cases cited8 are not in
point. None of them involved a taxpayer who had, as in the present case, obtained a final
judgment against third persons for reimbursement of payments made. In those cases, there
was either no legally enforceable right at all or such claimed right was still to be, or being,
litigated. On the other hand, the rule is that loss deduction will be denied if there is a
measurable right to compensation for the loss, with ultimate collection
reasonably clear. So where there is reasonable ground for reimbursement, the taxpayer
must seek his redress and may not secure a loss deduction until he establishes that no
recovery may be had.9 In other words, as the Tax Court put it, the taxpayer (petitioner)
must exhaust his remedies first to recover or reduce his loss. But assuming that there was
no reasonable expectation of recovery, still no loss deduction can be had. Sec. 30 (d) (2) of
the Tax Code requires a charge-off as one of the conditions for loss deduction: In the case
of a corporation, all losses actually sustained and charged-off within the taxable year and
not compensated for by
insurance or otherwise. Mertens12 states only four (4) requisites because the United States
Internal Revenue Code of 193913 has no charge-off requirement. Sec. 23(f) thereof
provides merely: In the case of a corporation, losses sustained during the taxable year and
not compensated for by insurance or otherwise. Petitioner, who had the burden of proof14
failed to adduce evidence that there was a charge-off in connection with the P44,490.00
or P30,600.00 which it paid to Galang Machinery.
CHINA BANKING CORPORATION V COURT OF APPEALS
Facts: Petitioners mad a 53% equity investment in First CBC Capital (Asia) Limited to the
amount of P16,227,851.80 consisting of 106,000 shares with par value of P100 per share.
Subsequently, First CBC was found to be insolvent. Petitioners, with the approval of the
BSP, wrote off as worthless its investment in the company and treated it as a bad debt or
ordinary loss deductible from its gross income. The Commissioner of Internal Revenue
disallowed the deduction saying that the investment could not be considered "worthless"
since First CBC could still exercise its financing and investment activities even if it was no
longer licensed as a depository. Even assuming that the securities had become worthless,

it still cannot be considered as a "bad debt" or expense since there is no indebtedness


between petitioner and First CBC. It should be classified as a "capital loss."
Held: The SC found in favor of respondents.
1. Not and indebtedness. An equity investment in shares of stock cannot be considered as
an indebtedness of First CBC Capital to China Bank. The former has no obligation to repay
the latter the amount invested. The amount China Bank invested in First CBC is, in fact, an
asset.
2. Capital asset, not ordinary. Capital assets are defined in the negative by Sec 33(1) of the
NIRC as property held by the TP exclusive of items primarily for the sale to customers in
the ordinary course of business, or property used in trade or business. Hence, securities,
such as equity holdings, are ordinary assets only in the hands of a dealer, or a person
actively engaged in trading in the same for his own account.
3. Section 29(d)(4)(B) of the NIRC treats the worthlessness of the securities held as
capital assets as a loss resulting from the sale or exchange of capital assets. Strictly
speaking, no sale occurs when securities held as capital assets become worthless.
Nonetheless, the law treats it as a loss from a sale just the same.
4. Section 33 of the NIRC provides that the capital loss sustained can only be deducted
from any capital gain derived within the taxable year. The same provision enumerates
assets which are not subject to the said limitation but equity holdings are not one of them.
THE CITY LUMBER, INC V DOMINGO
Facts: Respondent Domingo made an assessment on an additional income of petitioners
in the amount of P16, 678.63 coming from: 1. the sale of plywood, kegs of nails, and GI
sheets amouting to P 7,902.07, 2. a cash credit balance of P7,896.80 Petitioner assails the
validity of the assessment alleging that the inventory in question was not sold but were lost
to looters during a fire which occurred in the city. To this end, petitioner presented the
testimony of the Chief of Police of Dumaguete City affirming the occurrence of the fire.
Also, the petitioner claimed that the cash credit balance appearing in their books was
actually a loan secured by petitioners.
HELD:The assessments were valid. 1. The alleged loss of the plywood and kegs of nails
was never reported in the books of the petitioner nor in the petitioner's ITR for that year.
Such conduct of the petitioner proves that such loss never occurred. 2. Similarly, there was
neither any record in petitioner's books nor any receipt or other piece of evidence to show
receipt of the supposed loan.
MARCELO STEEL CROP V COLLECTOR OF INTERNAL REVENUE
Facts: RA 35 granted a four-year tax exemption from all internal revenue taxes to
enterprises, directly payable by such enterprise or person, which shall engage in new and
necessary industries. Petitioner corporation was engaged in: 1. the manufacture of wire
fences; 2. the manufacture of steel nails; 3. the manufacture of steel bars, rods, and other
allied products
of which the last two were covered by RA 35 In 1953 and 1954, petitioner filed its ITR
showing a net income of P34,386.58 and P58,329.00, respectively, derived solely from its
wire fence manufacturing business. It was accordingly assessed P12,750 in taxes which it
paid. subsequently, it filed an amended ITR for the same taxable years showing that it
actually incurred a loss of P871,407.37 and P104,956.29, respectively. The losses were
arrived at by consolidating the gross income and the gross allowable deductions of its three
industries. Petitioner thus filed for a refund of the P12,750 it initially paid in taxes on the
theory that since it is a corporation organized with a single capital to answer for all its
financial obligations, the gross income from both tax-exempt and non-exempt industries
and its liability should be based on the difference between its consolidated gross income
and its consolidated allowable deductions.
Held: Petition has no merit; Marcelo Steel cannot consolidate. 1. The purpose of RA 35 is
to encourage the establishment of new and necessary industries for the economic growth
of the country. In effect, it grants a subsidy to entrepreneurs who blaze a trail in a new
industry since there are greater risks involved in the same and an ROI is usually not
immediately forthcoming. As such, a tax exemption granted to an entrepreneur engaged in
a tax-exempt industry cannot be extended to benefit non-exempt industries in which the

same entrepreneur is concurrently engaged. The justification is simply not there since such
industries are, presumably, already deriving profits from its operations. 2. Single capital The fact that all three industries are organized under a single capital is of no moment. It is
clear that the law intended that tax exempt
and non-exempt industries be treated separately as reflected in EO 341, series 0f 1950,
which was incorporated in RA 901, a later incarnation of the same law.
PHILIPPINE SUGAR ESTATE CORP V COLLECTOR OF INTERNAL
REVENUE
Facts: Petitioner owned shares of stock in several different companies and 154 liberty
bonds at par value of P100 each. It received certain sums from said corporation
representing dividends on its shares of stock as well as interst on the said liberty bonds.
Petitioner filed its ITR inclusive
of the sums above-stated for which it was assessed an amount in taxes. It paid said
amount without protest. Subsequently, petitioner filed a demand for refund. In the letterdemand, the ground for the refund invoked was the illegality o the collection since the
dividends and interest were exempt from payment of income tax. Although in the present
petition, the ground invoked is that the income tax on dividends had already been paid at
the source. Respondent denied the claim for refund because plaintiff failed to lodge a
protest concurrently when it paid its tax liability as per Sec 1579 of the Revised
Administrative Code, as amended by Act No. 3685. Plaintiff
countered by saying that the applicable law is Act No. 2833 which requires no protest. In
addition, it invoked the ruling in Fox v Edwards wherein it was ruled that any unduly paid
income tax may be refunded without the necessity of a protest.
Held: The SC found in favor of the Collector.
1.Controlling law - There is no need to distinguish between the two laws in this case.
Section 19 of Act 2833 provides that "all administrative, special, and general provisions of
law, including laws in relation to the assessment, remission, collection, and refund of
internal revenue taxes
not hereto fore repealed and not inconsistent with...this Law are...applicable...to this Law."
By virtue of this saving clause, Section 1579 of the Revised Administrative Code finds
application to Act 2833.
2. Fox v Edwards is not controlling since the law specifically applied therein did not require
the lodging of a protest concurrently with the payment for a TP to retain such right to
protest. (NOTE fr digester: I did not find anything on losses or bad debts in this case. I may
have overlooked it.)
DEDUCTIONS AND EXEPTIONS; ALLOWABLE DEDUCTIONS; BAD
DEBTS
PHILIPPINE REFINING COMPANY V COURT OF APPEALS
Facts: In 1985, petitioner filed its ITR where it claimed 16 items amounting to P713,070.93
as bad debts and therefor deductible. Subsequently, the Commissioner for Internal
Revenue disallowed such deductions and assessed petitioner to pay a deficiency tax for
the year of 1985. Petitioner paid the deficiency tax under protest which the Commissioner
denied.
Upon a petition for review, the CTA modified the findings of the Commissioner by reducing
the deficiency tax assessment on the basis that three of the sixteen supposed bad debts
could be allowed as deductions. The CA later on agreed with the CTA.
Held: The SC upheld the ruling of the CA which it found to be in accordance with the SC's
ruling in Collector v Goodrich. It held the petitioner failed to substantiate the
worthlessness of the 13 debts which it claimed as deductions. As per the ruling in
Collector v Goodrich, to qualify as a bad debt, a TP must show: 1. that there is a valid and
subsisting debt;
2. that the debt must be actually ascertained to be worthless and uncollectible durring the
taxable year;
3. the debt must be charged off during the taxable year; and
4. the debt must arise from the business or trade of the TP.

In addition, the Court said, before a debt can be considered worthless, the TP must also
show that it is indeed uncollectible even in the future. Furthermore, the TP must undertake
several steps to prove that he exerted diligent efforts to collect the debt:
1. sending statements of accounts to the debtors; 2. sending of collection letters; 3. giving
the account to a lawyer for collection; and
4. filing a collection case in court. In the case at bar, the petitioner miserably failed to show
any of the foregoing. The only piece of evidence it offered to show the worthlessness of the
debts was the testimony of the company's financial adviser or accountant. The Court found
that this lacked the required probity to establish that the accounts it claimed as bad debts
were indeed worthless. Apart from such testimony, the petitioner failed to introduce even a
single iota of evidence to bolster its claim of worthlessness. (NOTE: In the rest of the case,
the Court presents the allegation of the petitioner as to why it could not collect on any of the
13 debts followed by
a statement how the petitioner failed to introduce evidence to substantiate such allegation.)
Collector of Internal Revenue v Goodrich International Rubber Co.
(21 SCRA 1336; No. L-22265)
Facts: In 1951 and 1952, respondent Goodrich filed it ITR in which it claimed an aggregate
amount (consisting of 18 individual accounts) of P50,455.41 as deductible for being bad
debts. The Collector of Internal Revenue disallowed the deductions and accordingly
assessed Goodrich accordingly. Goodrich protested the assessment and subsequently filed
an appeal
with the CTA which allowed the deductions for bad debts. Hence, this appeal by the
Government.
Held: Petition is partially meritorious. Some of the items claimed by Goodrich can rightfully
be written off as bad debts. The SC rejected the claim for deduction of 10 items because
Goodrich
failed to establish that that the debts were actually worthless or that it had reasonable
grounds to believe them to be so in 1951. The law permits the deduction of debts actually
ascertained to be worthless within the taxable year, obviously to prevent arbitrary action by
the TP to unduly avoid tax liability. Good faith on the part of the TP is not enough. He must
furthermore show that he had reasonably investigated the relevant facts and had drawn a
reasonable inference from the information thus obtained by him. At any rate, respondent
failed to prove that the debts were indeed worthless and that the debtors had no ability to
pay them. On the contrary, of these 10 accounts some payments were actually made
(some in full) after they had been characterized as bad debts and written off. The Court
however ruled that 8 of the 18 claimed bad debts can be
allowed as deductions. Common among these 8 was the action of Goodrich in persistently
demanding payment from its debtors; it's endorsement of the accounts to counsel for
collection; the pursuit of legal remedies for the collection on these debts; and the continuing
failure/clear inability of the debtors to pay off their obligations.
DEDUCTIONS AND EXEMPTIONS; ALLOWABLE DEDUCTIONS;
DEPRECIATION
BASILAN ESTATES, INC. v. CIR
Facts: Basilan Estates, Inc. claimed deductions for the depreciation of its assets on the
basis of their acquisition cost. As of January 1, 1950 it changed the depreciable value of
said assets by increasing it to conform with the increase in cost for their replacement.
Accordingly, from 1950 to
1953 it deducted from gross income the value of depreciation computed on the reappraised
value. CIR disallowed the deductions claimed by petitioner, consequently assessing the
latter of deficiency income taxes.
Issue:Whether or not the depreciation shall be determined on the acquisition cost rather
than the reappraised value of the assets.
Held: Yes. The following tax law provision allows a deduction from gross income for
depreciation but limits the recovery to the capital invested in the asset being depreciated:
(1)In general. A reasonable allowance for deterioration of property arising out of its use
or employment in the business or trade, or out of its not being used: Provided, That when

the allowance authorized under this subsection shall equal the capital invested by the
taxpayer . . . no further allowance shall be made. . . . The income tax law does not
authorize the depreciation of an asset beyond its acquisition cost. Hence, a deduction over
and above such cost cannot be claimed and allowed. The reason is that deductions from
gross income are privileges, not matters of right. They are not created by implication but
upon clear expression in the law. Depreciation is the gradual diminution in the useful value
of tangible property resulting from wear and tear and normal obsolescense. It commences
with the acquisition of the property and its owner is not bound to see his property gradually
waste, without making provision out of earnings for its replacement. The recovery, free of
income tax, of an amount more than the invested capital in an asset will transgress the
underlying purpose of a depreciation allowance. For then what the taxpayer would recover
will be, not only the acquisition cost, but also some profit. Recovery in due time thru
depreciation of investment made is the philosophy behind depreciation allowance; the idea
of profit on the investment made has never been the underlying reason for the allowance of
a deduction for depreciation.
Zamora v. CIR
Facts: These are 4 cases regarding deficiency income taxes allegedly incurred by the
Zamoras.
Cases Nos. L-15290 and L-15280:Mariano Zamora, owner of the Bay View Hotel and
Farmacia Zamora, Manila, filed his income tax returns the years 1951 and 1952. The
Collector of Internal Revenue found that he failed to file his return of the capital gains
derived from the sale of certain real properties and claimed deductions which were not
allowable. The CTA reduced the sum due Zamora and on appeal, petitioner alleged that the
CTA erred in disallowing the promotion expenses incurred by his wife for promotion of the
above businesses, depreciation of the Bayview Hotel Bldg, and in applying the Ballantyne
scale of values for determining the cost of his Manila property. The CIR, on the other hand,
claimed that the CTA erred in reducing the amounts and giving credence to the
uncorroborated testimony of Mariano Zamora that he bought the said real property in
question during the Japanese occupation, partly in Philippine currency and partly in
Japanese war notes. Cases Nos. L-15289 and L-15281 Mariano Zamora and his deceased
sister Felicidad Zamora, bought a piece of land located in Manila on May 16, 1944, for
P132,000.00 and sold it for P75,000.00 on March 5, 1951. They also purchased a lot
located inQuezon City for P68,959.00 on January 19, 1944, which they sold for P94,000 on
February 9, 1951. The CTA ordered the estate of the late Felicidad Zamora (represented by
Esperanza A. Zamora, as special administratrix of her estate), to pay the sum of P235.50,
representing alleged deficiency income tax and surcharge due from said estate.
First issue disallowance of the entire promotion expenses incurred by Mrs. Zamora
Petitioner: The CTA erred in disallowing P10,478.50 as promotion expenses incurred by
his wife for the promotion of the BayView Hotel and Farmacia Zamora. He contends that
the whole amount of P20,957.00 as promotion expenses should be allowed and not merely
one-half of it. on the ground that, while not all the itemized expenses are supported by
receipts, the absence of some supporting receipts has been sufficiently and satisfactorily
established - to purchase machinery for a new Tiki-Tiki plant, and to observe hotel
management in modern hotels.
Respondents: Mrs. Zamora obtained only the sum of P5,000.00 from the Central Bank
and that in her application for dollar allocation, she stated that she was going abroad on a
combined medical and business trip, which facts were not denied by Mariano Zamora. The
alleged expenses were not supported by receipts. Mrs. Zamora could not even remember
how much money she had when she left abroad in 1951, and how the alleged amount of
P20,957.00 was spent. There having been no means by which to ascertain which expense
was incurred by her in
connection with the business of Mariano Zamora and which was incurred for her personal
benefit, the respondents considered 50% of the said amount of P20,957.00 as business
expenses and the other 50%, as her personal expenses.
Held: The 50% allocation is very fair to Zamora, there being no receipt to explain the
alleged business expenses as well as the personal expenses that might have been
incurred. While in situations like the present, absolute certainty is usually no possible, the

CTA should make as close an approximation as it can, bearing heavily, if it chooses, upon
the taxpayer whose inexactness is of his own making. Section 30, of the Tax Code,
provides that in computing net income, there shall be allowed as deductions all the ordinary
and necessary expenses paid or incurred during the taxable year, in carrying on any trade
or business. Since promotion expenses constitute one of the deductions in conducting a
business, same must testify these requirements. Claim for the deduction of promotion
expenses or entertainment expenses must also be substantiated or supported by record
showing in detail the amount and nature of the expenses incurred.
Second issue disallowance/reduction of the rate of depreciation of Bayview Hotel
(from 3.5% to 2.5%)
Petitioner: Contends that 1) the Ermita district is becoming a commercial district, 2) the
hotel has no room for improvement, and(3) the changing modes in architecture, styles of
furniture and decorative designs, "must meet the taste of a fickle public". Also,the reference
to Bulletin F, a publication by the IRS, should have been first proved as law to be subject of
judicial notice.
Held: The CTA was approximately correct in holding that the rate of depreciation must be
2.5%. An average hotel buildings estimated useful life is 5 years, but inasmuch as it also
depends on the use and location, change in population and other, it is allowed a
deprecation rate of 2.5% which corresponds to a useful life of 40 years. It is true that
Bulletin F has no binding force, but it has a strong persuasive effect considering that the
same has been the result of scientific studies and observation for a long period in the
United States after whose Income Tax Law ours is patterned." Verily, courts are permitted
to look into and investigate the antecedents or the legislative history of the statutes
involved.
Third issue-the undeclared capital gains derived from the sales in 1951 of certain
real properties in Malate, Manila and in Quezon City, acquired during the Japanese
occupation.
Held: The CTAs appraisal in this case is correct. Consequently, the total undeclared
income of petitioners derived from the sales of the Manila and Quezon City properties in
1951 is P17,111.75 (P1,750.00 plus P15,361.75), 50% of which in the sum of P8,555.88 is
taxable, the said properties being capital assets held for more than one year. The cost
basis of property acquired in Japanese war notes is the equivalent of the war notes in
genuine Philippine currency in accordance with the Ballantyne Scale of values, and that the
determination of the gain derived or loss sustained in the sale of such property is not
affected by the decline at the time of sale, in the purchasing power of the Philippine
currency. It was found by the CTA that the purchase price of P132,000.00 was not entirely
paid in Japanese War notes but thereof or P66,000.00 was in Philippine currency. This
being the case, the Ballantyne Scale of values, which was the result
of an impartial scientific study, adopted and given judicial recognition, should be applied. As
the value of the Japanese war notes in May, 1944 when the Manila property was bought,
was 1 of the genuine Philippine Peso (Ballantyne Scale), and since the gain derived or
loss sustained in
the disposition of this property is to reckoned in terms of Philippine Peso, the value of the
Japanese war notes used in the purchase of the property, must be reduced in terms of the
genuine Philippine Peso to determine the cost of acquisition. It, therefore, results that since
the sum of P66,000.00 in Japanese war notes in May, 1944 is equivalent to P5,500.00 in
Philippine currency (P66,000.00 divided by 12), the acquisition cost of the property in
question is P66,000.00 plus P5,500.00 or P71,500.00 and that as the property was sold for
P75,000.00 in 1951, the owners thereof Mariano and Felicidad Zamora derived a capital
gain of P3,500.00or P1,750.00 each. For the Quezon City property, the CTA was correct in
giving credence to
Zamoras testimony that the same was purchased inPhilippine currency, because it is quite
incredible that real property with an assessed value of P46,910.00 should have been soldin
Japanese war notes with an equivalent value in Philippine currency of only P17,239.75.
Thus, the gain derived from the sale isP15,361.75, after deducting from the selling price the
cost of acquisition in the sum of P68,959.00 and the expense of sale in the sum of
P9,679.25.
Disposition: The petitions are dismissed, and the decision appealed from is affirmed.

DEDUCTIONS AND EXEMPTIONS; ALLOWABLE DEDUCTIONS;


DEPLETION
CONSOLIDATED MINES V CTA
Facts: Cosolidated Mines filed a refund for overpayments of income taxes for the year
1951. However, after investigation of the BIR, instead of having a refund, the company was
instead assessed for deficiency income taxes for the years 1953, 1954, and 1956, with 5%
surcharge
and 1% monthly interest. According to the investigation, (A) for the years 1951 to 1954 (1)
the Company had not accrued as an expense the share in the company profits of Benguet
Consolidated Mines as operator of the Consolidated's mines, although for income tax
purposes the Consolidated had reported income and expenses on the accrual basis; (2)
depletion and depreciation expenses had been overcharged; and (3) the claims for audit
and legal fees and miscellaneous expenses for 1953 and 1954 had not been properly
substantiated; and that (B) for the year 1956 (1) the Company had overstated its claim for
depletion; and (2) certain claims for miscellaneous expenses were not duly supported by
evidence Background info: Consolidated and Benguet entered into a development
agreement whereby Consolidated, as the owner of several mining claims, allowed Benguet
to explore, develop, mine, concentrate, and
market the ore in the mining claims. In the agreement, it was provided that benguet is to
provide the funds necessary for the expenses until such time the properties are on a profit
producing basis, to be reimbursed by consolidated. Once profit is derived, expenditures
from its own resources shall be charged against the subsequent gross income of the
properties. During the time Benguet is being reimbursed for all its expenditures, the net
profits resulting from the operation of the claims shall be divided 90% to Benguet and 10%
to Consolidated. Such division of net profits shall be based on the receipts, and
expenditures during each calendar year, and
shall continue until such time as the 90% of the net profits pertaining to Benguet hereunder
shall equal the amount of such expenditures. After Benguet has been fully reimbursed for
its expenditures, the net profits from the operation shall be divided between Benguet and
Consolidated share and share alike, it being understood however, that the net profits as the
term is used in this agreement shall be computed by deducting from gross income all
operating expenses and all disbursements of any nature whatsoever as may be made in
order to carry out the terms of this agreement. It appears that by 1953 Benguet had
completely recouped its advances, because they were then dividing the profits share and
share alike. Consolidated used the accrual method of accounting in computing its income.
One of its expenses is the amount-paid to Benguet as mine operator, which amount is
computed as 50% of net income. The
Consolidated deducts as an expense 50% of cash receipts minus disbursements, but does
not deduct at the end of each calendar year what the Commissioner alleges is "50% of the
share of Benguet" in the "accounts receivable." However, it deducts Benguet's 50% if and
when the "accounts receivable" are actually paid. It would seem, therefore, that
Consolidated has been deducting a portion of this expense (Benguet's share as mine
operator) on the "cash & carry" basis. The question is whether or not the accounting
system used by Consolidated justifies such a treatment of this item; and if not, whether said
method used by Consolidated, and characterized by the Commissioner as a "hybrid
method," may be allowed under the provisions of the NIRC.
Issue: WON Consolidateds accounting method is allowed
Held: YES. It is said that accounting methods for tax purposes comprise a set of rules for
determining when and how to report income and deductions. The U.S. Internal Revenue
Code allows each taxpayer to adopt the accounting method most suitable to his business,
and requires only that taxable income generally be based on the method of accounting
regularly employed in keeping the taxpayer's books, provided that the method clearly
reflects income.
A deduction cannot be accrued until an actual liability is incurred, even if payment has not
been made. Here we have to distinguish between (1) the method of accounting used by
Consolidated in determining its net income for tax purposes; and (2) the method of
computation agreed upon between Consolidated and Benguet in determining the amount

of compensation that was to be paid by the former to the latter. The parties, being free to do
so, had contracted that in the method of computing compensation the basis were "cash
receipts" and "cash payments." Once determined in accordance with the stipulated bases
and procedure, then the amount due Benguet for each month accrued at the end of that
month, whether Consolidated had made payment or not. To make Consolidated deduct as
an expense one-half of the "Accounts Receivable" would, in effect, be equivalent to giving
Benguet a right which it did not have under the contract, and to substitute for the parties'
choice a mode of computation of compensation
not contemplated by them.
ON DEPLETION:
The first issue raised by Consolidated is with respect to the rate of mine depletion used by
the Court of Tax Appeals. The Tax Code provides that in computing net income there shall
be allowed as deduction, in the case of mines, a reasonable allowance for depletion thereof
not to exceed the market value in the mine of the product thereof which has been mined
and sold during the year for which the return is made [Sec. 30(g) (1) (B)]. As an income tax
concept, depletion is wholly a creation of the statute solely a matter of legislative grace.
Hence, the taxpayer has the burden of justifying the allowance of any deduction claimed.
As in connection
with all other tax controversies, the burden of proof to show that a disallowance of depletion
by the Commissioner is incorrect or that an allowance made is inadequate is upon the
taxpayer, and this is true with respect to the value of the property constituting the basis of
the deduction. This burden-of-proof rule has been frequently applied and a value claimed
has been disallowed for lack of evidence. Here, SC considered the evidence presented
(testimony of Eligio Garcia
and the Report to Stockholders (which includes the Balance Sheet as of 1946), geological
report on the estimated amount of ore in the claims, etc.) it set forth a very detailed
computation of the depletion rate, determining the value of each component of the formula
of depletion, viz: Rate of Depletion Per Unit = Cost of Mine Property / Estimated ore
Deposit of Product Mined and sold
- depletion is different from depreciation In determining the amount of cost depletion
allowable the following three facts are essential, namely, (1) the basis of the property, (2)
the estimated total recoverable units in the property; and (3) the number of units recovered
during the taxable year in question. As used as an element in cost depletion, basis means
the dollar amount of the
taxpayer's capital or investment in the property which he is entitled to recover tax free
during the period he is removing the mineral in the deposit.
Disposition Decision modified
Footnotes in the case that are helpful: While taxable income is based on the method of
accounting used by the taxpayer, it will almost always differ from accounting income. This is
so
because of a fundamental difference in the ends the two concepts serve. Accounting
attempts to match cost against revenue. Tax law is aimed at collecting revenue. It is quick
to treat an item as income, slow to recognize deductions or losses. Thus, the tax law will
not recognize deductions for contingent future losses except in very limited situations.
Good accounting, on the other hand, their recognition. Once this fundamental difference in
approach is accepted, income tax accounting methods can be understood more easily. 33
Am. Jur. 2d 688. Under the accrual system income is accruable in the year in which the
taxpayer's right thereto becomes fixed and definite, even though it may not be actually
received until a later year, while a deduction for a liability is to be accrued and taken when
the liability becomes fixed and certain, even though it may not be paid until a later year. It
has been held that the basis of the accrual system of accounting is that obligations incurred
in the normal course of business will be discharged in due course; that the deductions have
been "paid or accrued" or "paid and incurred;" but in order to be accruable in the taxable
year, a valid obligation upon which the profit (or loss, in the case of a deduction) is to be
determined must have existed in the year in which the obligation became binding or
enforceable. The date of the accrued right to receive income, or the obligation to pay or
expend money constituting a deductible loss, is the date that fixes liability. Gain or loss may
not said to be fixed or accrued when the obligation is contingent upon the happening of a

future event. No duty or liability to pay an income tax upon a transaction arises until the
taxable year in which the event constituting the condition precedent occurs under any
system ofaccounting.
DEDUCTIONS AND EXEMPTIONS; ALLOWABLE DEDUCTIONS;
CHARITABLE AND OTHER CONTRIBUTIONS
ROXAS v. CTA, GR No L-25043, April 26, 1968
Facts: Don Pedro Roxas and Dona Carmen Ayala, Spanish subjects, transmitted to their
grandchildren by hereditary succession several properties. To manage the abovementioned properties, said children, namely, Antonio Roxas, Eduardo Roxas and Jose
Roxas, formed a
partnership called Roxas y Compania. At the conclusion of the WW2, the tenants who have
all been tilling the lands in Nasugbu for generations expressed their desire to purchase
from Roxas y Cia. the parcels which they actually occupied. For its part, the Government, in
consonance with the constitutional mandate to acquire big landed estates and apportion
them among landless tenants-farmers, persuaded the Roxas brothers to part with their
landholdings. Conferences were held with the farmers in the early part of 1948 and finally
the Roxas brothers agreed to sell 13,500 hectares to the Government for distribution to
actual occupants for a price of P2,079,048.47 plus P300,000.00 for survey and subdivision
expenses. It turned out however that the Government did not have funds to cover the
purchase price, and so a special arrangement was made for the Rehabilitation Finance
Corporation to advance to Roxas y Cia. The amount of P1,500,000.00 as loan. Collateral
for such loan were the lands proposed to be sold to the farmers. Under the arrangement,
Roxas y Cia. allowed the farmers to buy the lands for the same price but by installment,
and contracted with the Rehabilitation Finance Corporation
to pay its loan from the proceeds of the yearly amortizations paid by the farmers. The CIR
demanded from Roxas y Cia the payment of deficiency income taxes resulting from the
inclusion as income of Roxas y Cia. of the unreported 50% of the net profits for 1953 and
1955 derived from the sale of the Nasugbu farm lands to the tenants, and the disallowance
of deductions from gross income of various business expenses and contributions claimed
by Roxas y Cia. and the Roxas brothers. For the reason that Roxas y Cia. subdivided its
Nasugbu farm lands and sold
them to the farmers on installment, the Commissioner considered the partnership as
engaged in the business of real estate, hence, 100% of the profits derived therefrom was
taxed. The Roxas brothers protested the assessment but inasmuch as said protest was
denied, they instituted an appeal in the CTA which sustained the assessment. Hence, this
appeal.
Issue: Is Roxas y Cia. liable for the payment of deficiency income for the sale of Nasugbu
farmlands?
Held: NO. The proposition of the CIR cannot be favorably accepted in this isolated
transaction with its peculiar circumstances in spite of the fact that there were hundreds of
vendees. Although they paid for their respective holdings in installment for a period of 10
years, it would nevertheless not make the vendor Roxas y Cia. a real estate dealer during
the 10-year amortization period. It should be borne in mind that the sale of the Nasugbu
farm lands to the very farmers who tilled them for generations was not only in consonance
with, but more in obedience to the request and pursuant to the policy of our Government to
allocate lands to the landless. It was the bounden duty of the Government to pay the
agreed compensation after it had persuaded Roxas y Cia. to sell its haciendas, and to
subsequently subdivide them among the farmers at very reasonable terms and prices.
However, the Government could not comply with its duty for lack of funds. Obligingly,
Roxas y Cia. shouldered the Government's burden, went out of its way and sold lands
directly to the farmers in the same way and under the same terms as would have been the
case had the Government done it itself. For this magnanimous act, the municipal council of
Nasugbu passed a resolution expressing the people's gratitude.
In fine, Roxas y Cia. cannot be considered a real estate dealer for the sale in question.
Hence, pursuant to Section 34 of the Tax Code the lands sold to the farmers are capital
assets, and the gain derived from the sale thereof is capital gain, taxable only to the extent
of 50%.

DEDUCTIONS AND EXEMPTIONS; NON-DEDUCTIBLE EXPENSE


CIR v JAMIR
Facts: For the year 1954, Alberto M. K. Jamir declared a gross income of P75,858.65 and
claimed deductions aggregating P58,134.50, thereby showing a net income of P17,774.15,
upon which he paid P1,634 as income tax. The Collector of Internal Revenue, however,
assessed, as
deficiency income tax due from him, the sum of P16,395. Jamir appealed to the Court of
Tax Appeals, which reduced the amount due as deficiency income tax to P552.00
Issue: Whether Jamir had an undeclared income for the year 1954 aggregating
P31,274.91.
Held: No. It appears that, by using the so-called "expenditures method", the Government
considered as an undeclared income Jamir's expenditures for February and May were in
excess of his reported income for the same months. Although the Court of Tax Appeals, in
effect, sanctioned the adoption of the "expenditures method", it held that the same should
be applied by deducting the aggregate yearly expenditures from the declared yearly
income, not the expenditures incurred each month from the declared income therefor. In
the case at bar, Jamir's total income for the year 1954 (P75,858.65) exceeded (Pl7,774.15)
the total deductions (P58,134.50) claimed by him. Jamir introduced evidence that the said
sums of P1,281.24 and
P29,993.67 represented advances made to him by customers in the months of February
and may, 1954, and that the income derived from the corresponding transactions were
entered in his books of account in subsequent months, and this explanation was found by
the Court of Tax Appeals to have been proven satisfactorily. The next question raised by
appellant refers to Jamir's claim for car depreciation and salary of his driver. Although
petitioner had disallowed one-half (1/2) of these claims, it appearing that the car was used
by Jamir for personal and business purposes, the lower court allowed, as deductions,
three-fourths (3/4) of said amounts, the car having been used by Jamir "more for business
than for personal purpose". Petitioner
assails this as an error, but, considering the circumstances, we agree with the deduction of
by the lower court. It is next urged that Jamir committed fraud and the 50% surcharge
should not have been eliminated. But since Jamir did not have the undeclared income of
P31,274.91, upon which the contested assessment is mainly based, it follows necessarily
that he was not guilty
of the fraud and that the 50% surcharge has been properly eliminated.
Disposition: the decision appealed from is hereby affirmed.
Atlas Consolidated Mining v. CIR
Facts: CIR assessed Atlas deficiency income tax for 1957 to 1958 which amounted to
morethan P700K. CIR asserted that in 1957, Atlas was still not entitled to exemption
fromthe income tax under RA 909 because the same covers only gold mines and Atlas
isnot engaged in that. Atlas
protested before the Sec of Finance and Sec ruled that exemption provided in RA 909
embraces all new mines and old mines, whether gold or other minerals. Hence, Sec
recomputed and eliminated P500K+ in 1957 and reduced P215K to P39K in 1958. Atlas
appealed to this assessments assailing the disallowance of the following items: transfer
agents fees, stockholders relation service fee, US stock listing expenses, suit
expenses, provision for contingency. CTA disallowed the items except the stockholders
relation service fee and suit expenses. Also CTA ruled that the exemption from payment of
the corporate income tax of Atlas was good only up to the first quarter of 1958, hence it
computed for its net taxable income for the remaining of the year. Atlas appealed
asserting that the annual public relations expense is a deductible expense from gross
income because it is an ordinary and necessary
business expense.
Issue: WON this fee paid for the services rendered by a public relations firm in the US
labeled as stockholders relation service fee is an allowable deduction.
Held: No. it is a capital expenditure and not an ordinary expense. The principle is
recognized that when a taxpayer claims a deduction, he must point to some specific

provision of the statute in which that deduction is authorized and must be able to prove that
he is entitled to the deduction which the law allows. As previously adverted to, the law
allowing expenses as deduction from gross income for purposes of the income tax is
Section 30 (a) (1) of the National Internal Revenue which allows a deduction of "all the
ordinary and necessary expenses paid or
incurred during the taxable year in carrying on any trade or business." An item of
expenditure, in order to be deductible under this section of the statute, must fall squarely
within its language. We come, then, to the statutory test of deductibility where it is axiomatic
that to be deductible as a business expense, three conditions are imposed, namely: (1) the
expense must be ordinary and necessary, (2) it must be paid or incurred within the taxable
year, and (3) it must be paid or incurred in carrying in a trade or business. In addition, not
only must the taxpayer meet the
business test, he must substantially prove by evidence or records the deductions claimed
under the law, otherwise, the same will be disallowed. The mere allegation of the taxpayer
that an item of expense is ordinary and necessary does not justify its deduction. Assuming
that the expenditure is ordinary and necessary in the operation of the taxpayer's business,
the answer to the question as to whether the expenditure is an allowable deduction as a
business expense must be determined from the nature of the expenditure itself, which in
turn depends on the extent and permanency of the work accomplished by the expenditure.
The expenditure of P25,523.14 paid to P.K. Macker & Co. as compensation for services
carrying on the selling campaign in an effort to sell Atlas' additional capital stock of
P3,325,000 is not an ordinary
expense in line with the decision of U.S. Board of Tax Appeals in the case of Harrisburg
Hospital Inc. vs. Commissioner of Internal Revenue. Accordingly, as found by the Court of
Tax Appeals, the said expense is not deductible from Atlas gross income in 1958 because
expenses relating to 1) recapitalization and reorganization of the corporation, 2) the cost of
obtaining stock subscription 3)promotion expenses and 4) commission or fees paid for the
sale of stock reorganization are capital expenditures. The burden of proof that the
expenses incurred are ordinary and necessary is onthe taxpayer. The claimed business
expense must also be supported by appropriate documents such as invoices, official
receipts, and contracts, to be made available in case of a tax audit by the Bureau of
Internal Revenue.
GANCAYCO V COLLECTOR
Facts:
Gancayco filed his Income tax Return (ITR) for 1949.
CIR notified him that his liability is Php 9.793.62, which he paid 1950
CIR after a year wrote to Gancayco saying that there was tax due from him for a total of
Php 29,554.05
Gancayco asked for reconsideration and the tax assessed wasreduced
CIR issued a warrant of distraint for the deficient liability
Gancayco filed petition with CTA
CTA: Required Gancayco to pay Php 16, 860.31 for tax deficiency in1949
Gancayco: the right to collect the deficiency income tax is barred by thestatute of
limitations.
: the 5 yr period for judicial action should be counted from May 12 50, the date of original
assessment SC: Section 316 provides: The civil remedies for the collection of internal
revenue taxes, fees, or charges, and any increment thereto resulting from delinquency shall
be (a) by distraint of goods, chattels, or effects, and other personal property of whatever
character, including stocks and other securities, debts, credits, bank accounts, and interest
in and rights to personal property, and by levy upon real property; and (b) by judicial action.
Either of these remedies or both simultaneously may be pursued in the discretion of the
authorities charged with the collection of such taxes. No exemption shall be allowed against
the internal revenue taxes in any case.
: Deduction for expenses may be allowed, however in this case, Gancayco was not able to
prove any expense as there were no receipts or other proofs. CTA AFFIRMED
DEDUCTIONS AND EXEMPTIONS; NON-DEDUCTIBLE EXPENSES;

ILLEGAL EXPENSES
CALANOC V COLLECTOR
Facts:
Calanoc was authorized to solicit and receive contributions for the orphans and destitute
kids of the Child Welfare Workers Club of the Social Welfare Commission.
Dec 1949, Calanoc financed and promoted a boxing exhibition at the Rizal Memorial
Stadium for said charitable purpose
He applied for exemption from payment of the amusement tax as provided in Sec 260
NIRC
CIR investigated the tax case of Calanoc and it was found that there was gross sale of
Php 26,553, expenditure of 25,157 and profit of 1,375.30
Profit was remitted to Social Welfare Commission.
CIR demanded Calanoc oto pay 533
Sec of Finance denied the application of Calanoc for exemption from payment of
amusement tax
CTA: Affirmed the assessment of 7k Calanoc; denies receving a stadium fee of 1k
: Although it was shown that 1k was paid by O-OSO Beverages
: His accountant is dead
SC: the items of expenditures for deduction are exorbitant and not supported by receipts
CTA Affirmed
3M PHILIPPINES, INC., petitioner, vs. COMMISSIONER OF
INTERNAL REVENUE, respondent. [G.R. No. 82833. September 26,
1988.]
Facts: The petitioner claimed as deductions for income tax purposes "business expenses"
in the form of royalty payments to its foreign licensor which the respondent Commissioner
of Internal Revenue disallowed. The petitioner claimed the following deductions royalties
and technical service fees and pre-operational cost of tape coater. The amount was not
allowed as entire deduction. The petitioner argues that the law applicable to its case is only
Section 29(a)(1) of the Tax Code and not Circular No. 393 of the Central Bank.
Issue: WON the royalty payments are valid deductible expense. WON the Tax Code is
applicable.
Held: No. Although the Tax Code allows payments of royalty to be deducted from gross
income as business expenses, it is CB Circular No. 393 that defines what royalty payments
are proper. Improper payments of royalty are not deductible as legitimate business
expenses. Section 3-c of CB Circular No. 393 provides for payment of royalties only on
commodities manufactured by the licensee under the royalty agreement not on the
wholesale price of finished products imported by the licensee from the licensor. entral Bank
Circulars, like CB Circular No. 393 (dated December 7, 1973, published in the Official
Gazette issue of December 17, 1973 [69 O.G. No. 51, p. 11737] issued by the Central Bank
in the exercise of fits authority under the Central Bank Act, duly published in the Official
Gazette, have the force and effect of law (Cases cited) and
binding on everybody.
DEDUCTIONS AND EXEMPTIONS; PERSONAL EXEMPTIONS
COLLECTOR OF INTERNAL REVENUE, petitioner, vs. ORLANDO V.
CALSADO and COURT OF TAX APPEALS, respondents. [G.R. No. L10293. February 27, 1959.]
Facts: The appellant refused to recognize the appellee as head of a family within the
meaning of section (b) of the National Internal Revenue Code (Commonwealth Act No. 466,
as amended by Republic Act No. 590) and assessed him for deficiency taxes. According to
the respondent-appellee he has a brother below 21 years old mainly dependent upon him
for support.

Issue: WON the respondent is classified as head of the family.


Held: Yes. For an unmarried individual to fall within the term "head of a family" under
Section 23 (b) of the National Internal Revenue Code, it is enough that he has either of the
following who is dependent upon him for his chief support: a father or mother or both, one
or more brothers or sisters, one or more legitimate, recognized natural or adopted children,
provided such brother, sister or child is less than 21 years of age. The fact that the father is
still alive and continues to exercise parental authority over his minor children is of no
moment. All that the law requires in order that an unmarried individual may be considered
as head of a family is that the relatives enumerated be dependent upon him for their chief
support. *Under RA 9504 Regardless of the classification, the allowed personal exemption
is Php50,000.
In the matter of the adoption of the minor MARCIAL ELEUTERIO RESABA. LUIS
SANTOS-YIGO and LIGIA MIGUEL DE SANTOSYIGO, petitioners-appellees, vs.
REPUBLIC OF THE PHILIPPINES, oppositor-appellant.TAX ON CORPORATIONS:
BASES AND RATES;
RESIDENT FOREIGN CORPORATIONS; TAXABLE INCOME [G.R.
No. L-6294. June 28, 1954.]
Facts: The petitioners adopted a child while they have two legitimate children of their own.
The said children were born after the agreement for adoption was executed by petitioners
and the parents of the minor.
Issue: WON the adoption is valid
Held: Yes. The purpose of adoption is to afford to persons who have no child of their own
the consolation of having one by creating, through legal fiction, the relation of paternity and
filiation where none exists by blood relationship. This purpose rejects the idea of adoption
by persons who have children of their own, for otherwise, conflicts, friction, and differences
may arise resulting from the infiltration of foreign element into a family which already counts
with children upon whom the parents can shower their paternal love and affection. While
the adoption agreement was executed at a time when the law applicable to adoption is
Rule 100 of the Rules of Court, which does not prohibit persons who have legitimate
children from adopting, such
agreement can not have the effect of establishing the relation of paternity and filiation by
fiction of law without the sanction of court. The only valid adoption in this jurisdiction is that
one made through court, or in pursuance of the procedure laid down by the rule. *For tax
purposes - A head of family is an individual who actually supports and maintains in one
household one or more individuals, who are closely connected with him by blood
relationship, relationship by
marriage, or by adoption, and whose right to exercise family control and provide for these
dependent individuals is based upon some moral or legal obligation. In the absence of
continuous actual residence together, whether or not a person with dependent relatives is a
head of a family within the meaning of the statute must depend on the character of the
separation. If a father is absent on business, or a child or other dependent is away at
school or on a visit, the common home being still maintained, the additional exemption
applies. If, moreover, through force of circumstances a parent is obliged to maintain his
dependent children
with relatives or in a boarding house while he lives elsewhere, the additional exemption
may still apply. If, however, without necessity, the dependent continuously makes his home
elsewhere, his benefactor is not the head of a family, irrespective of the question of
support. A resident alien with children abroad is not thereby entitled to credit as the head of
a family. Chief support means principal or main support. Partial support not amounting to
chief support will not entitle the taxpayer to claim exemption as a head of a family.
Regulation No. section 11

You might also like